ASA 5505 to allow 2nd network segment through mpls

I have been having a heck of a time trying to configure my 5505 to allow the second segment on my network to use the internet.
Office 1 has a fiber internet connection, and all traffic flows fine.
Office 2 had gotten it's internet from AT&T, via a network based firewall injecting a default route into the mpls cloud.
both offices connunicate to each other through the mpls.
When we added the fiber to office 1, we had the mpls people change the default internet route to the inside address of the 5505 and things worked fine.
when AT&T attempted to remove the NBF defaut route, and inject the 5505's address as default, things didn't go so well.
AT&T claims that it is within my nat cmmands on the 5505, but won't tell me anything else.  I assume that they are correct, and I assume that I am not good enough with the 5505 ASDM to tell it what to do.
Office 1 uses 10.10.30.xx addresses and Office 2 uses 10.10.10.xx - the 5505 inside interface is 10.10.30.2 the internal interfaces of the mpls are 10.10.30.1 and 10.10.10.1
I don't know what other information you would need, but am stuck here at Office 1 until I can get this working.
Thanks

Hi,
Ok, so IF I have not understood anything wrong (which is still possible ), it would seem to me that the network mask of the ASA is atleast one reason that will cause problems for WI LAN if they try to use the Internet through the ASA5505 on the PA site.
This is what I would presume will happen when a host on the WI LAN initiates a connection to the Internet
WI PC 10.10.10.10 sends a TCP SYN to initiate/open a TCP connection with a Web server on the Internet
The TCP SYN gets forwarded to the default gateway of the PC which is 10.10.10.1
The TCP SYN packet traverses the ISP MPLS network all the way to the PA Site
The PA Site 3900 has a default route probably towards PA ASA 10.10.30.2
TCP SYN gets forwarded from the PA 3900 to the PA ASA according to the above mentioned default route on the PA 3900
TCP SYN arrives on the ASA and gets forwarded to the Internet
TCP SYN,ACK from the Web server arrives on the ASA
ASA will ARP for the MAC address of the WI PC IP address of 10.10.10.10 because it thinks that the host is directly connected to the ASAs "inside" interface because of the "inside" interfaces large /16 network mask which contains addresses between 10.10.0.0 - 10.10.255.255
The ARP request sent from the ASA never receives a reply since the WI PC isnt directly connected
PA ASA will never be able to forward the traffic to the WI PC which is trying to open the connection to the Internet because of the above mentioned problem. Therefore the TCP connection from WI PC never succeeds and timeouts.
Now you might ask, why does the connections between the PA and WI LAN work. To my understanding is that because the traffic from the PA hosts gets first forwarded to the PA 3900 then they have a working route to the WI LAN. The same way the WI LAN has a working route towards the PA LAN since the ASA isnt not involed in anyway.
The PA Internet connection naturally works as the 10.10.30.0/24 hosts are directly connected to the ASA so the above mentioned ARP will not fail on their part and traffic is forwarded just fine between the PA LAN and the Internet.
So to my understanding the solution to this problem would be to change the PA ASA "inside" subnet mask from 255.255.0.0 to 255.255.255.0.
If you are unsure of the of this change I would suggest you do it when there is low network use (so you can revernt the change) Naturally if you are on the PA LAN then you can probably access the Console connection if something were to go wrong. I cant see any configurations on the PA ASA which would imply that you configure the device remotely through the Internet.
Hope I made sense and hope this helps
Naturally ask more if needed
- Jouni

Similar Messages

  • Cisco ASA 5505 Routing between internal networks

    Hi,
    I am new to Cisco ASA and have been configuring my new firewall but one thing have been bothering. I cannot get internal networks and routing between them to work as I would like to. Goal is to set four networks and control access with ACL:s between those.
    1. Outside
    2. DMZ
    3. ServerNet1
    4. Inside
    ASA version is 9.1 and i have been reading on two different ways on handling IP routing with this. NAT Exempt and not configuring NAT at all and letting normal IP routing to handle internal networks. No matter how I configure, with or without NAT I cannot get access from inside network to DMZ or from ServerNet1 to DMZ. Strange thing is that I can access services from DMZ to Inside and ServerNet1 if access list allows it. For instance DNS server is on Inside network and DMZ works great using it.
    Here is the running conf:
    interface Ethernet0/0
    switchport access vlan 20
    interface Ethernet0/1
    switchport access vlan 20
    interface Ethernet0/2
    switchport access vlan 19
    interface Ethernet0/3
    switchport access vlan 10
    switchport trunk allowed vlan 10,19-20
    switchport trunk native vlan 1
    interface Ethernet0/4
    switchport access vlan 10
    interface Ethernet0/5
    switchport access vlan 10
    switchport trunk allowed vlan 10-11,19-20
    switchport trunk native vlan 1
    switchport mode trunk
    interface Ethernet0/6
    switchport access vlan 10
    switchport trunk allowed vlan 10-11,19-20
    switchport trunk native vlan 1
    switchport mode trunk
    interface Ethernet0/7
    switchport access vlan 10
    interface Vlan10
    nameif inside
    security-level 90
    ip address 192.168.2.1 255.255.255.0
    interface Vlan11
    nameif ServerNet1
    security-level 100
    ip address 192.168.4.1 255.255.255.0
    interface Vlan19
    nameif DMZ
    security-level 10
    ip address 192.168.3.1 255.255.255.0
    interface Vlan20
    nameif outside
    security-level 0
    ip address dhcp setroute
    ftp mode passive
    clock timezone EEST 2
    clock summer-time EEDT recurring last Sun Mar 3:00 last Sun Oct 4:00
    object network obj_any
    subnet 0.0.0.0 0.0.0.0
    object network obj-192.168.2.0
    subnet 192.168.2.0 255.255.255.0
    object network obj-192.168.3.0
    subnet 192.168.3.0 255.255.255.0
    object network DNS
    host 192.168.2.10
    description DNS Liikenne
    object network Srv2
    host 192.168.2.10
    description DC, DNS, DNCP
    object network obj-192.168.4.0
    subnet 192.168.4.0 255.255.255.0
    object network ServerNet1
    subnet 192.168.4.0 255.255.255.0
    object-group protocol TCPUDP
    protocol-object udp
    protocol-object tcp
    object-group network RFC1918
    object-group network InternalNetworks
    network-object 192.168.2.0 255.255.255.0
    network-object 192.168.3.0 255.255.255.0
    object-group service DM_INLINE_SERVICE_1
    service-object tcp destination eq domain
    service-object udp destination eq domain
    service-object udp destination eq nameserver
    service-object udp destination eq ntp
    object-group service DM_INLINE_TCP_1 tcp
    port-object eq www
    port-object eq https
    port-object eq ftp
    port-object eq ftp-data
    object-group service rdp tcp-udp
    description Microsoft RDP
    port-object eq 3389
    object-group service DM_INLINE_TCP_2 tcp
    port-object eq ftp
    port-object eq ftp-data
    port-object eq www
    port-object eq https
    object-group service DM_INLINE_SERVICE_2
    service-object tcp destination eq domain
    service-object udp destination eq domain
    object-group network DM_INLINE_NETWORK_1
    network-object object obj-192.168.2.0
    network-object object obj-192.168.4.0
    access-list dmz_access_in extended permit ip object obj-192.168.3.0 object obj_any
    access-list dmz_access_in extended deny ip any object-group InternalNetworks
    access-list DMZ_access_in extended permit object-group TCPUDP object obj-192.168.3.0 object DNS eq domain
    access-list DMZ_access_in extended permit object-group TCPUDP object obj-192.168.3.0 object-group DM_INLINE_NETWORK_1 object-group rdp
    access-list DMZ_access_in extended deny ip any object-group InternalNetworks
    access-list DMZ_access_in extended permit tcp object obj-192.168.3.0 object obj_any object-group DM_INLINE_TCP_2
    access-list inside_access_in extended permit ip object obj-192.168.2.0 object-group InternalNetworks
    access-list inside_access_in extended permit object-group TCPUDP object obj-192.168.2.0 object obj_any object-group rdp
    access-list inside_access_in extended permit tcp object obj-192.168.2.0 object obj_any object-group DM_INLINE_TCP_1
    access-list inside_access_in extended permit object-group DM_INLINE_SERVICE_1 object Srv2 object obj_any
    access-list inside_access_in extended permit object-group TCPUDP object obj-192.168.2.0 object obj-192.168.3.0 object-group rdp
    access-list ServerNet1_access_in extended permit object-group DM_INLINE_SERVICE_2 any object DNS
    access-list ServerNet1_access_in extended permit ip any any
    pager lines 24
    logging enable
    logging asdm informational
    mtu ServerNet1 1500
    mtu inside 1500
    mtu DMZ 1500
    mtu outside 1500
    no failover
    icmp unreachable rate-limit 1 burst-size 1
    asdm image disk0:/asdm-711-52.bin
    no asdm history enable
    arp timeout 14400
    no arp permit-nonconnected
    nat (inside,DMZ) source static obj-192.168.2.0 obj-192.168.2.0 destination static obj-192.168.2.0 obj-192.168.2.0 no-proxy-arp
    object network obj_any
    nat (inside,outside) dynamic interface
    nat (DMZ,outside) after-auto source dynamic obj_any interface destination static obj_any obj_any
    nat (ServerNet1,outside) after-auto source dynamic obj-192.168.4.0 interface
    access-group ServerNet1_access_in in interface ServerNet1
    access-group inside_access_in in interface inside
    access-group DMZ_access_in in interface DMZ
    timeout xlate 3:00:00
    timeout pat-xlate 0:00:30
    timeout conn 1:00:00 half-closed 0:10:00 udp 0:02:00 icmp 0:00:02
    timeout sunrpc 0:10:00 h323 0:05:00 h225 1:00:00 mgcp 0:05:00 mgcp-pat 0:05:00
    timeout sip 0:30:00 sip_media 0:02:00 sip-invite 0:03:00 sip-disconnect 0:02:00
    timeout sip-provisional-media 0:02:00 uauth 0:05:00 absolute
    timeout tcp-proxy-reassembly 0:01:00
    timeout floating-conn 0:00:00
    dynamic-access-policy-record DfltAccessPolicy
    user-identity default-domain LOCAL
    aaa authentication ssh console LOCAL
    http server enable
    http 192.168.2.0 255.255.255.0 inside
    http 192.168.4.0 255.255.255.0 ServerNet1
    no snmp-server location
    no snmp-server contact
    snmp-server enable traps snmp authentication linkup linkdown coldstart warmstart
    crypto ipsec security-association pmtu-aging infinite
    crypto ca trustpool policy
    telnet timeout 5
    ssh 192.168.4.0 255.255.255.0 ServerNet1
    ssh 192.168.2.0 255.255.255.0 inside
    ssh timeout 5
    console timeout 0
    dhcpd auto_config outside
    threat-detection basic-threat
    threat-detection statistics access-list
    no threat-detection statistics tcp-intercept
    class-map inspection_default
    match default-inspection-traffic
    policy-map type inspect dns preset_dns_map
    parameters
      message-length maximum client auto
      message-length maximum 512
    policy-map global_policy
    class inspection_default
      inspect dns preset_dns_map
      inspect ftp
      inspect h323 h225
      inspect h323 ras
      inspect rsh
      inspect rtsp
      inspect esmtp
      inspect sqlnet
      inspect skinny
      inspect sunrpc
      inspect xdmcp
      inspect sip
      inspect netbios
      inspect tftp
      inspect ip-options
      inspect icmp
    service-policy global_policy global
    prompt hostname context
    no call-home reporting anonymous

    Hi Jouni,
    Yep, Finnish would be good also =)
    In front of ASA is DSL modem, on the trunk ports is Hyper-V host that uses the trunk ports so that every VM has their VLAN ID defined in the VM level. Everything is working good on that end. Also there is WLAN Access Pois on one of the ASA ports, on the WLAN AP there is the management portal address on DMZ that i have been testing agains (192.168.3.4)
    If i configure Dynamic PAT from inside to the DMZ then the traffic starts to work from inside to all hosts on DMZ but thats not the right way to do it so no shortcuts =)
    Here is the conf now, still doesnt work:
    interface Ethernet0/0
    switchport access vlan 20
    interface Ethernet0/1
    switchport access vlan 20
    interface Ethernet0/2
    switchport access vlan 19
    interface Ethernet0/3
    switchport access vlan 10
    switchport trunk allowed vlan 10,19-20
    switchport trunk native vlan 1
    interface Ethernet0/4
    switchport access vlan 10
    interface Ethernet0/5
    switchport access vlan 10
    switchport trunk allowed vlan 10-11,19-20
    switchport trunk native vlan 1
    switchport mode trunk
    interface Ethernet0/6
    switchport access vlan 10
    switchport trunk allowed vlan 10-11,19-20
    switchport trunk native vlan 1
    switchport mode trunk
    interface Ethernet0/7
    switchport access vlan 10
    interface Vlan10
    nameif inside
    security-level 90
    ip address 192.168.2.1 255.255.255.0
    interface Vlan11
    nameif ServerNet1
    security-level 100
    ip address 192.168.4.1 255.255.255.0
    interface Vlan19
    nameif DMZ
    security-level 10
    ip address 192.168.3.1 255.255.255.0
    interface Vlan20
    nameif outside
    security-level 0
    ip address dhcp setroute
    ftp mode passive
    clock timezone EEST 2
    clock summer-time EEDT recurring last Sun Mar 3:00 last Sun Oct 4:00
    object network obj_any
    subnet 0.0.0.0 0.0.0.0
    object network obj-192.168.2.0
    subnet 192.168.2.0 255.255.255.0
    object network obj-192.168.3.0
    subnet 192.168.3.0 255.255.255.0
    object network DNS
    host 192.168.2.10
    description DNS Liikenne
    object network Srv2
    host 192.168.2.10
    description DC, DNS, DNCP
    object network obj-192.168.4.0
    subnet 192.168.4.0 255.255.255.0
    object network ServerNet1
    subnet 192.168.4.0 255.255.255.0
    object-group protocol TCPUDP
    protocol-object udp
    protocol-object tcp
    object-group network RFC1918
    object-group network InternalNetworks
    network-object 192.168.2.0 255.255.255.0
    network-object 192.168.3.0 255.255.255.0
    object-group service DM_INLINE_SERVICE_1
    service-object tcp destination eq domain
    service-object udp destination eq domain
    service-object udp destination eq nameserver
    service-object udp destination eq ntp
    object-group service DM_INLINE_TCP_1 tcp
    port-object eq www
    port-object eq https
    port-object eq ftp
    port-object eq ftp-data
    object-group service rdp tcp-udp
    description Microsoft RDP
    port-object eq 3389
    object-group service DM_INLINE_TCP_2 tcp
    port-object eq ftp
    port-object eq ftp-data
    port-object eq www
    port-object eq https
    object-group service DM_INLINE_SERVICE_2
    service-object tcp destination eq domain
    service-object udp destination eq domain
    object-group network DM_INLINE_NETWORK_1
    network-object object obj-192.168.2.0
    network-object object obj-192.168.4.0
    object-group network DEFAULT-PAT-SOURCE
    description Default PAT source networks
    network-object 192.168.2.0 255.255.255.0
    network-object 192.168.3.0 255.255.255.0
    network-object 192.168.4.0 255.255.255.0
    access-list dmz_access_in extended permit ip object obj-192.168.3.0 object obj_any
    access-list dmz_access_in extended deny ip any object-group InternalNetworks
    access-list DMZ_access_in extended permit object-group TCPUDP object obj-192.168.3.0 object DNS eq domain
    access-list DMZ_access_in extended permit object-group TCPUDP object obj-192.168.3.0 object-group DM_INLINE_NETWORK_1 object-group rdp
    access-list DMZ_access_in extended deny ip any object-group InternalNetworks
    access-list DMZ_access_in extended permit tcp object obj-192.168.3.0 object obj_any object-group DM_INLINE_TCP_2
    access-list inside_access_in extended permit ip object obj-192.168.2.0 object-group InternalNetworks
    access-list inside_access_in extended permit object-group TCPUDP object obj-192.168.2.0 object obj_any object-group rdp
    access-list inside_access_in extended permit tcp object obj-192.168.2.0 object obj_any object-group DM_INLINE_TCP_1
    access-list inside_access_in extended permit object-group DM_INLINE_SERVICE_1 object Srv2 object obj_any
    access-list inside_access_in extended permit object-group TCPUDP object obj-192.168.2.0 object obj-192.168.3.0 object-group rdp
    access-list ServerNet1_access_in extended permit object-group DM_INLINE_SERVICE_2 any object DNS
    access-list ServerNet1_access_in extended permit ip any any
    pager lines 24
    logging enable
    logging asdm informational
    mtu ServerNet1 1500
    mtu inside 1500
    mtu DMZ 1500
    mtu outside 1500
    no failover
    icmp unreachable rate-limit 1 burst-size 1
    asdm image disk0:/asdm-711-52.bin
    no asdm history enable
    arp timeout 14400
    no arp permit-nonconnected
    nat (any,outside) after-auto source dynamic DEFAULT-PAT-SOURCE interface
    access-group ServerNet1_access_in in interface ServerNet1
    access-group inside_access_in in interface inside
    access-group DMZ_access_in in interface DMZ
    timeout xlate 3:00:00
    timeout pat-xlate 0:00:30
    timeout conn 1:00:00 half-closed 0:10:00 udp 0:02:00 icmp 0:00:02
    timeout sunrpc 0:10:00 h323 0:05:00 h225 1:00:00 mgcp 0:05:00 mgcp-pat 0:05:00
    timeout sip 0:30:00 sip_media 0:02:00 sip-invite 0:03:00 sip-disconnect 0:02:00
    timeout sip-provisional-media 0:02:00 uauth 0:05:00 absolute
    timeout tcp-proxy-reassembly 0:01:00
    timeout floating-conn 0:00:00
    dynamic-access-policy-record DfltAccessPolicy
    user-identity default-domain LOCAL
    aaa authentication ssh console LOCAL
    http server enable
    http 192.168.2.0 255.255.255.0 inside
    http 192.168.4.0 255.255.255.0 ServerNet1
    no snmp-server location
    no snmp-server contact
    snmp-server enable traps snmp authentication linkup linkdown coldstart warmstart
    crypto ipsec security-association pmtu-aging infinite
    crypto ca trustpool policy
    telnet timeout 5
    ssh 192.168.4.0 255.255.255.0 ServerNet1
    ssh 192.168.2.0 255.255.255.0 inside
    ssh timeout 5
    console timeout 0
    dhcpd auto_config outside
    threat-detection basic-threat
    threat-detection statistics access-list
    no threat-detection statistics tcp-intercept
    class-map inspection_default
    match default-inspection-traffic
    policy-map type inspect dns preset_dns_map
    parameters
      message-length maximum client auto
      message-length maximum 512
    policy-map global_policy
    class inspection_default
      inspect dns preset_dns_map
      inspect ftp
      inspect h323 h225
      inspect h323 ras
      inspect rsh
      inspect rtsp
      inspect esmtp
      inspect sqlnet
      inspect skinny
      inspect sunrpc
      inspect xdmcp
      inspect sip
      inspect netbios
      inspect tftp
      inspect ip-options
      inspect icmp
    service-policy global_policy global
    prompt hostname context
    no call-home reporting anonymous

  • Cisco ASA 5505 and PAT

    So I have a weird problem that I'm hoping someone has a point in the right direction I can follow... At home I have a Cisco ASA 5505 - not very complex network some BCP configs and it's providing a NAT (PAT). I have a static IP and using a few RFC 1918 segments - like I said nothing earth shattering. I have a linksys E1200 802.11N WPA2 PSK - again pretty standard. I connect laptops, iPads, iPhones, Kindles, Androids no problem. Until recently my 60" Vizio had no issues using the network (wired or wireless). Now network is failing on the TV. I see it get to the FW and I can ping trace etc... to the TV. The FW logs show resets (log is below).
    Now here is the real interesting part - if I turn the tether feature on my iPhone on and connect the TV to it - it works - what's even more interesting is if I then go back to the home network it all works again no problem until I reboot the TV... HELP!
    Apr 19 15:34:09 192.168.10.254 %ASA-6-305011: Built dynamic TCP translation from inside:10.10.10.139/60657 to outside:68.162.222.142/57003
    Apr 19 15:34:09 192.168.10.254 %ASA-6-302013: Built outbound TCP connection 61988 for outside:98.137.204.251/443 (98.137.204.251/443) to inside:10.10.10.139/60657 (68.162.222.142/57003)
    Apr 19 15:34:09 192.168.10.254 %ASA-6-302014: Teardown TCP connection 61988 for outside:98.137.204.251/443 to inside:10.10.10.139/60657 duration 0:00:00 bytes 3689 TCP Reset-I
    Apr 19 15:34:12 192.168.10.254 %ASA-6-305011: Built dynamic TCP translation from inside:10.10.10.139/60658 to outside:68.162.222.142/53332
    Apr 19 15:34:12 192.168.10.254 %ASA-6-302013: Built outbound TCP connection 61989 for outside:98.137.204.251/443 (98.137.204.251/443) to inside:10.10.10.139/60658 (68.162.222.142/53332)
    Apr 19 15:34:12 192.168.10.254 %ASA-6-305011: Built dynamic TCP translation from inside:10.10.10.139/37006 to outside:68.162.222.142/40015
    Apr 19 15:34:12 192.168.10.254 %ASA-6-302013: Built outbound TCP connection 61990 for outside:98.136.10.32/443 (98.136.10.32/443) to inside:10.10.10.139/37006 (68.162.222.142/40015)
    Apr 19 15:34:12 192.168.10.254 %ASA-6-302014: Teardown TCP connection 61989 for outside:98.137.204.251/443 to inside:10.10.10.139/60658 duration 0:00:00 bytes 3689 TCP Reset-I
    Apr 19 15:34:12 192.168.10.254 %ASA-6-302014: Teardown TCP connection 61990 for outside:98.136.10.32/443 to inside:10.10.10.139/37006 duration 0:00:00 bytes 3689 TCP FINs
    A

    Hello ras,
    As you mentioned the TV is sending a reset packet to the remote address. I will recommend you to create a capture of the traffic and review the traffic at the packet level to see a posible reason for the drop.
    Here is how. Then you can download it to pcap format and uploaded to the forum for further analysis.
    http://www.cisco.com/c/en/us/support/docs/security/asa-5500-x-series-next-generation-firewalls/110117-asa-capture-asdm-config.html
    http://www.cisco.com/c/en/us/td/docs/security/asa/syslog-guide/syslogs/logmsgs.html#pgfId-6941209
    Hope this information is helpful.

  • ASA 5505 9.1 and NAT issues to single dynamic IP

    Good afternoon everybody, 
    a few days ago I tried setting up my ASA 5505 to allow access from the outside network to an Exchange server (ports HTTPS and SMTP) in my inside LAN.
    Everything seems to be working... until my outside IP address changes (for example due to a router reset or a disconnection caused by the ISP). 
    As soon as the outside address changes the NAT rules are deleted and these 2 lines pop up in the syslog :
    <166>%ASA-6-305012: Teardown static TCP translation from inside:192.168.1.150/25 to outside:79.6.105.13/25 duration 0:01:17.
    <166>%ASA-6-305012: Teardown static TCP translation from inside:192.168.1.150/443 to outside:79.6.105.13/443 duration 0:01:17.
    In the same time, the consolle connection shows these two messages :
    Asa5505# ERROR: NAT unable to reserve ports.
    ERROR: NAT unable to reserve ports.
    I have moved both Anyconnect VPN essentials and http ports to 10443 and 8080 respectively so port 443 should be free for nat.
    This is the configuration file, I  have marked the lines related to network objects and relative nat statements, I hope it helps to find out where's the problem.
    Obviously the lines in red are the ones disappearing... I'm quite desperate, actually.
    ASA Version 9.1(5) 
    hostname Asa5505
    domain-name home
    enable password XXXXXX encrypted
    names
    interface Ethernet0/0
     description ADSLPPoE
     switchport access vlan 2
    interface Ethernet0/1
     description Internal_LAN
    interface Ethernet0/2
     description Management_Net 
     switchport access vlan 3
    interface Ethernet0/3
     shutdown
    interface Ethernet0/4
     shutdown
    interface Ethernet0/5
     description Uplink
     switchport trunk allowed vlan 1,3
     switchport trunk native vlan 1
     switchport mode trunk
    interface Ethernet0/6
     description Wireless-POE
     switchport trunk allowed vlan 1,3
     switchport trunk native vlan 1
     switchport mode trunk
    interface Ethernet0/7
     description Webcam-POE 
    interface Vlan1
     nameif inside
     security-level 100
     ip address 192.168.1.250 255.255.255.0 
    interface Vlan2
     nameif outside
     security-level 0
     pppoe client vpdn group AliceADSL
     ip address pppoe setroute 
    interface Vlan3
     no forward interface Vlan1
     nameif management
     security-level 100
     ip address 10.5.1.250 255.255.255.0 
    ftp mode passive
    clock timezone CEST 1
    clock summer-time CEDT recurring last Sun Mar 2:00 last Sun Oct 3:00
    dns domain-lookup inside
    dns domain-lookup outside
    dns server-group DefaultDNS
     name-server 192.168.1.4
     domain-name home
    object network Exchange-HTTPS
     host 192.168.1.150
    object network Exchange-SMTP
     host 192.168.1.150
    object network Network_Inside
     subnet 192.168.1.0 255.255.255.0
    object network Network_Management
     subnet 10.5.1.0 255.255.255.0
    access-list Outside_ACL extended permit tcp any object Exchange-HTTPS eq https 
    access-list Outside_ACL extended permit tcp any object Exchange-SMTP eq smtp 
    pager lines 24
    logging enable
    logging asdm warnings
    mtu inside 1500
    mtu outside 1492
    mtu management 1500
    no failover
    icmp unreachable rate-limit 1 burst-size 1
    no asdm history enable
    arp timeout 14400
    no arp permit-nonconnected
    object network Exchange-HTTPS
     nat (inside,outside) static interface service tcp https https 
    object network Exchange-SMTP
     nat (inside,outside) static interface service tcp smtp smtp 
    object network Network_Inside
     nat (inside,outside) dynamic interface
    object network Network_Management
     nat (management,outside) dynamic interface
    access-group Outside_ACL in interface outside
    timeout xlate 3:00:00
    timeout pat-xlate 0:00:30
    timeout conn 1:00:00 half-closed 0:10:00 udp 0:02:00 icmp 0:00:02
    timeout sunrpc 0:10:00 h323 0:05:00 h225 1:00:00 mgcp 0:05:00 mgcp-pat 0:05:00
    timeout sip 0:30:00 sip_media 0:02:00 sip-invite 0:03:00 sip-disconnect 0:02:00
    timeout sip-provisional-media 0:02:00 uauth 0:05:00 absolute
    timeout tcp-proxy-reassembly 0:01:00
    timeout floating-conn 0:00:00
    dynamic-access-policy-record DfltAccessPolicy
    user-identity default-domain LOCAL
    http server enable 8080
    http 10.5.1.0 255.255.255.0 management
    no snmp-server location
    no snmp-server contact
    snmp-server enable traps snmp authentication linkup linkdown coldstart warmstart
    crypto ipsec security-association pmtu-aging infinite
    crypto ca trustpool policy
    telnet timeout 5
    ssh stricthostkeycheck
    ssh timeout 5
    ssh key-exchange group dh-group1-sha1
    console timeout 0
    management-access management
    vpdn group AliceADSL request dialout pppoe
    vpdn group AliceADSL localname aliceadsl
    vpdn group AliceADSL ppp authentication pap
    vpdn username aliceadsl password ***** store-local
    dhcpd address 192.168.1.100-192.168.1.130 inside
    dhcpd dns 192.168.1.4 192.168.1.150 interface inside
    dhcpd wins 192.168.1.4 interface inside
    dhcpd enable inside
    dhcpd address 10.5.1.30-10.5.1.40 management
    dhcpd dns 208.67.222.222 208.67.220.220 interface management
    dhcpd enable management
    threat-detection statistics access-list
    no threat-detection statistics tcp-intercept
    webvpn
     port 10443
     anyconnect-essentials
    class-map inspection_default
     match default-inspection-traffic
    policy-map type inspect dns preset_dns_map
     parameters
      message-length maximum client auto
      message-length maximum 512
    policy-map global_policy
     class inspection_default
      inspect dns preset_dns_map 
      inspect ftp 
      inspect h323 h225 
      inspect h323 ras 
      inspect ip-options 
      inspect netbios 
      inspect rsh 
      inspect rtsp 
      inspect skinny  
      inspect esmtp 
      inspect sqlnet 
      inspect sunrpc 
      inspect tftp 
      inspect sip  
      inspect xdmcp 
    service-policy global_policy global
    prompt hostname context 
    no call-home reporting anonymous
    call-home
     profile CiscoTAC-1
      no active
      destination address http https://tools.cisco.com/its/service/oddce/services/DDCEService
      destination address email [email protected]
      destination transport-method http
      subscribe-to-alert-group diagnostic
      subscribe-to-alert-group environment
      subscribe-to-alert-group inventory periodic monthly
      subscribe-to-alert-group configuration periodic monthly
      subscribe-to-alert-group telemetry periodic daily
    Cryptochecksum:XXXXXXXX
    : end
    no asdm history enable
    Thanks in advance for your precious help !
    C.

    Update 29th of June :
    Tried both suggestions: flashing to 9.22 didn't fix the problem. The only significant change between 9.1(5) and 9.2(2) is that as soon as I reload the configuration after a connection drop both nat rules are restored. In 9.1(5) the nat statements were removed from the runnning configuration when the PPPoE connection was lost, and the config was updated (or maybe saved?), so after a reload those statements were gone and I had to copy-paste them back in conf-t in order to restore them.
    I tried using show xlate both before, during, and after the connection drop. As expected before the disconnection of PPPoE the static PAT rules are there, and the dynamic ones as well. During disconnection, all the xlate table is clean empty and the aforementioned error "Asa5505# ERROR: NAT unable to reserve ports. ERROR: NAT unable to reserve ports." pops up in the terminal. After a few minutes (needed by the DSL modem to perform its reset and bring up the DSL line again) the connection is established once more, but the only rules appearing in xlate are the ones created by the dynamic statements for management and LAN. If i reload the ASA using reload noconfirm every rule is restored and everything works again.
    Two brief questions :
    1) in my NAT statements for PAT, does it change anything if I modify them (for example) from 
    nat (inside,outside) static interface service tcp https https
    to
    nat (inside,outside) dynamic interface service tcp https https 
    ? Since it seems like the dynamic PAT is restored after a connection drop I was asking myself what happens if I change the rules this way.
    2) if there's not any ohter way to fix this, is it possible to schedule a reload of the ASA as soon as the PPPoE connection drops in order to make this problem "self fixing" ? I can't predict how many times a day the line drops and I can't be there 24/7 with my consolle cable connected in order to restore the nat statements ^^
    Thank you for your precious help and patience !
    C.

  • Cisco ASA 5505 AnyConnect SSL VPN problem

    Hi!
    I have a small network, wiht ASA 5505, 8.4:
    Inside network: 192.168.2.0/24
    Outside: Static IP
    I would like to deploy a SSL AnyConnect setup.
    The state:
    -I give the correct IP from my predefined VPN pool (10.10.10.0/24).
    But, could not reach any resource, could not ping too. My host has given 10.10.10.1 IP, and I had a GW: 10.10.10.2. Where is this GW from?
    Could you help me?
    Here is my config (I omitted my PUBLIC IP, and GW): 
    Result of the command: "show running-config"
    : Saved
    ASA Version 8.4(4)1
    hostname valamiASA
    domain-name valami.local
    enable password OeyyCrIqfUEmzen8 encrypted
    passwd 2KFQnbNIdI.2KYOU encrypted
    names
    interface Ethernet0/0
    switchport access vlan 2
    interface Ethernet0/1
    interface Ethernet0/2
    interface Ethernet0/3
    interface Ethernet0/4
    interface Ethernet0/5
    interface Ethernet0/6
    interface Ethernet0/7
    switchport access vlan 12
    interface Vlan1
    description LAN
    no forward interface Vlan12
    nameif inside
    security-level 100
    ip address 192.168.2.1 255.255.255.0
    interface Vlan2
    description WAN
    nameif outside
    security-level 0
    ip address MY_STATIC_IP 255.255.255.248
    interface Vlan12
    description Vendegeknek a valamiHotSpot WiFi-hez
    nameif guest
    security-level 100
    ip address 192.168.4.1 255.255.255.0
    management-only
    ftp mode passive
    clock timezone GMT 0
    dns domain-lookup inside
    dns domain-lookup outside
    dns domain-lookup guest
    dns server-group DefaultDNS
    name-server 62.112.192.4
    name-server 195.70.35.66
    domain-name valami.local
    same-security-traffic permit intra-interface
    object network obj_any
    subnet 0.0.0.0 0.0.0.0
    object network inside-net
    subnet 192.168.2.0 255.255.255.0
    object network guest-net
    subnet 192.168.3.0 255.255.255.0
    object network NETWORK_OBJ_192.168.2.128_25
    subnet 192.168.2.128 255.255.255.128
    object-group protocol DM_INLINE_PROTOCOL_3
    protocol-object ip
    protocol-object icmp
    object-group protocol DM_INLINE_PROTOCOL_1
    protocol-object ip
    protocol-object icmp
    object-group protocol DM_INLINE_PROTOCOL_2
    protocol-object ip
    protocol-object icmp
    access-list global_access extended permit object-group DM_INLINE_PROTOCOL_3 any any
    access-list AnyConnect_Client_Local_Print extended deny ip any any
    access-list AnyConnect_Client_Local_Print extended permit tcp any any eq lpd
    access-list AnyConnect_Client_Local_Print remark IPP: Internet Printing Protocol
    access-list AnyConnect_Client_Local_Print extended permit tcp any any eq 631
    access-list AnyConnect_Client_Local_Print remark Windows' printing port
    access-list AnyConnect_Client_Local_Print extended permit tcp any any eq 9100
    access-list AnyConnect_Client_Local_Print remark mDNS: multicast DNS protocol
    access-list AnyConnect_Client_Local_Print extended permit udp any host 224.0.0.251 eq 5353
    access-list AnyConnect_Client_Local_Print remark LLMNR: Link Local Multicast Name Resolution protocol
    access-list AnyConnect_Client_Local_Print extended permit udp any host 224.0.0.252 eq 5355
    access-list AnyConnect_Client_Local_Print remark TCP/NetBIOS protocol
    access-list AnyConnect_Client_Local_Print extended permit tcp any any eq 137
    access-list AnyConnect_Client_Local_Print extended permit udp any any eq netbios-ns
    access-list inside_access_in extended permit object-group DM_INLINE_PROTOCOL_2 any any
    access-list outside_access_in extended permit object-group DM_INLINE_PROTOCOL_1 any any
    pager lines 24
    logging enable
    logging asdm informational
    mtu inside 1500
    mtu outside 1500
    mtu guest 1500
    ip local pool valami_vpn_pool 10.10.10.1-10.10.10.10 mask 255.255.255.0
    icmp unreachable rate-limit 1 burst-size 1
    icmp permit any inside
    no asdm history enable
    arp timeout 14400
    object network inside-net
    nat (inside,outside) dynamic interface
    object network guest-net
    nat (guest,outside) dynamic interface
    access-group inside_access_in in interface inside
    access-group outside_access_in in interface outside
    access-group global_access global
    route outside 0.0.0.0 0.0.0.0 MY_STATIC_GW 1
    timeout xlate 3:00:00
    timeout pat-xlate 0:00:30
    timeout conn 1:00:00 half-closed 0:10:00 udp 0:02:00 icmp 0:00:02
    timeout sunrpc 0:10:00 h323 0:05:00 h225 1:00:00 mgcp 0:05:00 mgcp-pat 0:05:00
    timeout sip 0:30:00 sip_media 0:02:00 sip-invite 0:03:00 sip-disconnect 0:02:00
    timeout sip-provisional-media 0:02:00 uauth 0:05:00 absolute
    timeout tcp-proxy-reassembly 0:01:00
    timeout floating-conn 0:00:00
    dynamic-access-policy-record DfltAccessPolicy
    user-identity default-domain LOCAL
    aaa local authentication attempts max-fail 16
    http server enable
    http 192.168.2.0 255.255.255.0 inside
    no snmp-server location
    no snmp-server contact
    snmp-server enable traps snmp authentication linkup linkdown coldstart warmstart
    telnet timeout 5
    ssh timeout 5
    ssh key-exchange group dh-group1-sha1
    console timeout 0
    dhcpd auto_config outside
    threat-detection basic-threat
    threat-detection statistics host
    threat-detection statistics access-list
    no threat-detection statistics tcp-intercept
    webvpn
    enable inside
    enable outside
    anyconnect-essentials
    anyconnect image disk0:/anyconnect-win-2.5.2014-k9.pkg 1
    anyconnect enable
    tunnel-group-list enable
    group-policy GroupPolicy_valami_VPN internal
    group-policy GroupPolicy_valami_VPN attributes
    wins-server value 192.168.2.2
    dns-server value 192.168.2.2
    vpn-tunnel-protocol ssl-client
    split-tunnel-policy tunnelall
    default-domain value valami.local
    webvpn
      anyconnect ssl rekey time 30
      anyconnect ssl rekey method ssl
      anyconnect ask enable default anyconnect timeout 30
      customization none
      deny-message value Login was successful, but because certain criteria have not been met or due to some specific group policy, you do not have permission to use any of the VPN features. Contact your IT administrator for more information.
    username test password P4ttSyrm33SV8TYp encrypted
    tunnel-group valami_VPN type remote-access
    tunnel-group valami_VPN general-attributes
    address-pool valami_vpn_pool
    default-group-policy GroupPolicy_valami_VPN
    tunnel-group valami_VPN webvpn-attributes
    group-alias valami_VPN enable
    class-map inspection_default
    match default-inspection-traffic
    policy-map type inspect dns preset_dns_map
    parameters
      message-length maximum client auto
      message-length maximum 512
    policy-map global_policy
    class inspection_default
      inspect dns preset_dns_map
      inspect ftp
      inspect h323 h225
      inspect h323 ras
      inspect rsh
      inspect rtsp
      inspect esmtp
      inspect sqlnet
      inspect skinny 
      inspect sunrpc
      inspect xdmcp
      inspect sip 
      inspect netbios
      inspect tftp
      inspect ip-options
    service-policy global_policy global
    prompt hostname context
    no call-home reporting anonymous
    Cryptochecksum:d54de340bb6794d90a9ee52c69044753
    : end

    First of all thanks your link.
    I know your notes, but i don't understand 1 thing:
    if i check nat exemption in the anyconnect wizad, why should i make nat exemption rule?
    A tried creating a roule, but it is wrong.
    My steps (on ASDM):
    1: create network object (10.10.10.0/24), named VPN
    2: create nat rule: source any, destination VPN, protocol any
    Here is my config:
    Result of the command: "show running-config"
    : Saved
    ASA Version 8.4(4)1
    hostname companyASA
    domain-name company.local
    enable password OeyyCrIqfUEmzen8 encrypted
    passwd 2KFQnbNIdI.2KYOU encrypted
    names
    interface Ethernet0/0
    switchport access vlan 2
    interface Ethernet0/1
    interface Ethernet0/2
    interface Ethernet0/3
    interface Ethernet0/4
    interface Ethernet0/5
    interface Ethernet0/6
    interface Ethernet0/7
    switchport access vlan 12
    interface Vlan1
    description LAN
    no forward interface Vlan12
    nameif inside
    security-level 100
    ip address 192.168.2.1 255.255.255.0
    interface Vlan2
    description WAN
    nameif outside
    security-level 0
    ip address 77.111.103.106 255.255.255.248
    interface Vlan12
    description Vendegeknek a companyHotSpot WiFi-hez
    nameif guest
    security-level 100
    ip address 192.168.4.1 255.255.255.0
    ftp mode passive
    clock timezone CEST 1
    clock summer-time CEDT recurring last Sun Mar 2:00 last Sun Oct 3:00
    dns domain-lookup inside
    dns domain-lookup outside
    dns domain-lookup guest
    dns server-group DefaultDNS
    name-server 62.112.192.4
    name-server 195.70.35.66
    domain-name company.local
    same-security-traffic permit intra-interface
    object network obj_any
    subnet 0.0.0.0 0.0.0.0
    object network inside-net
    subnet 192.168.2.0 255.255.255.0
    object network guest-net
    subnet 192.168.3.0 255.255.255.0
    object network NETWORK_OBJ_192.168.2.128_25
    subnet 192.168.2.128 255.255.255.128
    object network WEBSHOP
    host 192.168.2.2
    object network INSIDE_HOST
    host 10.100.130.5
    object network VOIP_management
    host 192.168.2.215
    object network Dev_1
    host 192.168.2.2
    object network Dev_2
    host 192.168.2.2
    object network RDP
    host 192.168.2.2
    object network Mediasa
    host 192.168.2.17
    object network VOIP_ePhone
    host 192.168.2.215
    object network NETWORK_OBJ_192.168.4.0_28
    subnet 192.168.4.0 255.255.255.240
    object network NETWORK_OBJ_10.10.10.8_29
    subnet 10.10.10.8 255.255.255.248
    object network VPN
    subnet 10.10.10.0 255.255.255.0
    object network VPN-internet
    subnet 10.10.10.0 255.255.255.0
    object-group protocol DM_INLINE_PROTOCOL_3
    protocol-object ip
    protocol-object icmp
    object-group protocol DM_INLINE_PROTOCOL_1
    protocol-object ip
    protocol-object icmp
    object-group protocol DM_INLINE_PROTOCOL_2
    protocol-object ip
    protocol-object icmp
    access-list global_access extended permit object-group DM_INLINE_PROTOCOL_3 any any
    access-list AnyConnect_Client_Local_Print extended deny ip any any
    access-list AnyConnect_Client_Local_Print extended permit tcp any any eq lpd
    access-list AnyConnect_Client_Local_Print remark IPP: Internet Printing Protocol
    access-list AnyConnect_Client_Local_Print extended permit tcp any any eq 631
    access-list AnyConnect_Client_Local_Print remark Windows' printing port
    access-list AnyConnect_Client_Local_Print extended permit tcp any any eq 9100
    access-list AnyConnect_Client_Local_Print remark mDNS: multicast DNS protocol
    access-list AnyConnect_Client_Local_Print extended permit udp any host 224.0.0.251 eq 5353
    access-list AnyConnect_Client_Local_Print remark LLMNR: Link Local Multicast Name Resolution protocol
    access-list AnyConnect_Client_Local_Print extended permit udp any host 224.0.0.252 eq 5355
    access-list AnyConnect_Client_Local_Print remark TCP/NetBIOS protocol
    access-list AnyConnect_Client_Local_Print extended permit tcp any any eq 137
    access-list AnyConnect_Client_Local_Print extended permit udp any any eq netbios-ns
    access-list inside_access_in extended permit object-group DM_INLINE_PROTOCOL_2 any any
    access-list outside_access_in extended permit object-group DM_INLINE_PROTOCOL_1 any any
    pager lines 24
    logging enable
    logging asdm informational
    mtu inside 1500
    mtu outside 1500
    mtu guest 1500
    ip local pool company_vpn_pool 10.10.10.10-10.10.10.15 mask 255.255.255.0
    icmp unreachable rate-limit 1 burst-size 1
    icmp permit any inside
    no asdm history enable
    arp timeout 14400
    nat (any,any) source static any any destination static VPN VPN
    nat (inside,outside) source static inside-net inside-net destination static VPN VPN
    object network inside-net
    nat (inside,outside) dynamic interface
    object network guest-net
    nat (guest,outside) dynamic interface
    access-group inside_access_in in interface inside
    access-group outside_access_in in interface outside
    access-group global_access global
    route outside 0.0.0.0 0.0.0.0 77.111.103.105 1
    timeout xlate 3:00:00
    timeout pat-xlate 0:00:30
    timeout conn 1:00:00 half-closed 0:10:00 udp 0:02:00 icmp 0:00:02
    timeout sunrpc 0:10:00 h323 0:05:00 h225 1:00:00 mgcp 0:05:00 mgcp-pat 0:05:00
    timeout sip 0:30:00 sip_media 0:02:00 sip-invite 0:03:00 sip-disconnect 0:02:00
    timeout sip-provisional-media 0:02:00 uauth 0:05:00 absolute
    timeout tcp-proxy-reassembly 0:01:00
    timeout floating-conn 0:00:00
    dynamic-access-policy-record DfltAccessPolicy
    user-identity default-domain LOCAL
    aaa local authentication attempts max-fail 16
    http server enable
    http 192.168.2.0 255.255.255.0 inside
    no snmp-server location
    no snmp-server contact
    snmp-server enable traps snmp authentication linkup linkdown coldstart warmstart
    telnet timeout 5
    ssh timeout 5
    ssh key-exchange group dh-group1-sha1
    console timeout 0
    dhcpd auto_config outside
    threat-detection basic-threat
    threat-detection statistics host
    threat-detection statistics access-list
    no threat-detection statistics tcp-intercept
    webvpn
    enable inside
    enable outside
    anyconnect-essentials
    anyconnect image disk0:/anyconnect-win-2.5.2014-k9.pkg 1
    anyconnect enable
    tunnel-group-list enable
    group-policy GroupPolicy_company_VPN internal
    group-policy GroupPolicy_company_VPN attributes
    wins-server value 192.168.2.2
    dns-server value 192.168.2.2
    vpn-tunnel-protocol l2tp-ipsec
    split-tunnel-policy tunnelall
    default-domain value company.local
    webvpn
      anyconnect ssl rekey time 30
      anyconnect ssl rekey method ssl
      anyconnect ask enable default anyconnect timeout 30
      customization none
      deny-message value Login was successful, but because certain criteria have not been met or due to some specific group policy, you do not have permission to use any of the VPN features. Contact your IT administrator for more information.
    group-policy GroupPolicy_VPN internal
    group-policy GroupPolicy_VPN attributes
    wins-server none
    dns-server value 62.112.192.4 195.70.35.66
    vpn-tunnel-protocol ssl-client
    default-domain value company.local
    username test password P4ttSyrm33SV8TYp encrypted
    tunnel-group company_VPN type remote-access
    tunnel-group company_VPN general-attributes
    address-pool company_vpn_pool
    default-group-policy GroupPolicy_company_VPN
    tunnel-group company_VPN webvpn-attributes
    group-alias company_VPN enable
    tunnel-group VPN type remote-access
    tunnel-group VPN general-attributes
    address-pool company_vpn_pool
    default-group-policy GroupPolicy_VPN
    tunnel-group VPN webvpn-attributes
    group-alias VPN enable
    class-map inspection_default
    match default-inspection-traffic
    policy-map type inspect dns preset_dns_map
    parameters
      message-length maximum client auto
      message-length maximum 512
    policy-map global_policy
    class inspection_default
      inspect dns preset_dns_map
      inspect ftp
      inspect h323 h225
      inspect h323 ras
      inspect rsh
      inspect rtsp
      inspect esmtp
      inspect sqlnet
      inspect skinny 
      inspect sunrpc
      inspect xdmcp
      inspect sip 
      inspect netbios
      inspect tftp
      inspect ip-options
      inspect icmp
      inspect icmp error
    service-policy global_policy global
    prompt hostname context
    no call-home reporting anonymous
    Cryptochecksum:33ee37a3722f228f9be9b84ef43f731e
    : end
    Could you give me a CLI-code?
    (or ASDM steps).

  • Cisco ASA 5505 VPN Routing/Networking Question

    I have a very basic question about Cisco ASA 5505 IPsec Site to Site VPNs.  I want to install a Cisco ASA 5505 at a Data Center, in a LAN subnet that utilizes publicly routable IP addresses.  I would like to install a second Cisco ASA 5505 in a remote branch office as its peer. 
    Regardless of whether I use publicly routable IPs at the branch office in the "inside" network or non-routable IPs, how would the devices and servers at the Data Center know to route IP packets destined for the branch office back through the Cisco ASA instead of through the default gateway at the Data Center?  I can see accomplishing this if every single device at the Data Center is configured with routing table entries, but that isn't feasible.  It also isn't feasible to use the Cisco ASA 5505 as the default gateway for all of the devices as the Data Center, allowing it to decide where the traffic should go.
    What am I missing?  Is the solution to try to map branch office IPs to IP addresses within the Data Center's LAN subnet so that all of the traffic is on the same subnet?

    You can do it in several different ways.
    One way is to tell the server that if it has traffic to network x then it needs to go to the ASA all other traffic is to head for the default gateway.
    In windows this is done via the route command
    do not forget to make it "persistent" otherwise the route will disapear when your reboot the server.
    in unix/linux
    It is also the route command
    Or you can tell your "default gateway" to route that network to the ASA
    Good luck
    HTH

  • ASA 5505 as hw vpn client to PIX501 or ASA5505 w network extension mode

    Hi!
    We have been using a PIX 501 for a couple of years now to access a
    local network with Cisco VPN software client. However we now need
    access from another site with multiple users so I decided to buy two
    ASA 5505 UL bundle to do the job. First i tried to just hook up the
    new ASA at the remote site and connect to the PIX 501 with easy vpn.
    In went fine. I configured the new ASA right from the box with the old
    vpn profile settings and it worked right away. But as we also need the
    remote site to be accessed from the main site (PIX side) i tried to
    enable "network extension mode" but then the tunnel didnt work
    anymore. it connects but no traffic is coming through. I set it back
    to normal mode (only client) and it worked again.
    Is there anything else I need to do to be able to use network
    extension mode than just enabling it in ASDM ?
    The samt thing happens when using two ASA 5505 the same way.
    Software versions are:
    PIX: 6.3
    ASA 5505: 7.2.1 (used to be 7.2.2 but I had to downgrade because of a bug in 7.2.2 - vpnclient fails after reboot)
    I also did try the latest 8.2 with very little success. Seemed a bit buggy.
    Thanks,
    Bjorn

    Hi!
    Thought I could add some info. Our Head unit is 192.168.1.1 and the connecting ASA 5505 is 192.168.10.1. When I try to ping a machine (192.168.1.201) from the remote site I get this in the ASA log:
    With network extension mode
    302020 192.168.1.201 192.168.10.2 Built ICMP connection for faddr 192.168.1.201/0 gaddr 192.168.10.2/512 laddr 192.168.10.2/512
    With only client mode
    302020 192.168.1.201 192.168.10.2 Built ICMP connection for faddr 192.168.1.201/0 gaddr 192.168.1.9/1 laddr 192.168.10.2/512
    It seemes to me (quite the newbie here on ASA) that the unit does not handle the gateway address correctly when using network extension mode. The PC i use to ping from is 192.168.10.2.
    Any ideas from the experts ?
    Regards,
    B

  • Cisco ASA 5505 L2TP Pass through

    I am having trouble with L2TP pass through on an ASA 5505 device.
    L2TP server: OSX 10.6
    I can connect with any OSX system and it works fine straight away.
    When connecting with a windows computer I get a 789 error.  "Error 789: The L2TP connection attempt failed because the security layer encountere a processing error during the initial negotiations with the remote computer."
    I did not setup or configure the device to start with and apart from this issue its working fine so I am hessitant at trying to just mess around too much to try and find the problem.
    I am using the ASDM 6.4 to manage the device.
    Ports look to be forwarded correctly; 1701, 4500 & 500 UDP.
    Im just looking for other common issues?
    Rob

    Below is the commands you wanted.
    Where you see: IPNOTWHATIWASEXPECTING
    This is an IP I dont know. possible and old IP address.
    and
    default-domain value domain-notcorrect.local
    This is an old domain from years ago.
    Result of the command: "show run crypto"
    crypto ipsec transform-set aes-sha esp-aes esp-sha-hmac
    crypto ipsec transform-set aes-192-sha esp-aes-192 esp-sha-hmac
    crypto ipsec transform-set aes-256-sha esp-aes-256 esp-sha-hmac
    crypto ipsec transform-set 3des-sha esp-3des esp-sha-hmac
    crypto ipsec transform-set ESP-AES-128-SHA esp-aes esp-sha-hmac
    crypto ipsec transform-set ESP-AES-128-MD5 esp-aes esp-md5-hmac
    crypto ipsec transform-set ESP-AES-192-SHA esp-aes-192 esp-sha-hmac
    crypto ipsec transform-set ESP-AES-192-MD5 esp-aes-192 esp-md5-hmac
    crypto ipsec transform-set ESP-AES-256-SHA esp-aes-256 esp-sha-hmac
    crypto ipsec transform-set ESP-AES-256-MD5 esp-aes-256 esp-md5-hmac
    crypto ipsec transform-set ESP-3DES-SHA esp-3des esp-sha-hmac
    crypto ipsec transform-set ESP-3DES-MD5 esp-3des esp-md5-hmac
    crypto ipsec transform-set ESP-DES-SHA esp-des esp-sha-hmac
    crypto ipsec transform-set ESP-DES-MD5 esp-des esp-md5-hmac
    crypto ipsec transform-set transform-amzn esp-aes esp-sha-hmac
    crypto ipsec security-association lifetime seconds 28800
    crypto ipsec security-association lifetime kilobytes 4608000
    crypto dynamic-map map-dynamic 1 set pfs group5
    crypto dynamic-map map-dynamic 1 set transform-set aes-256-sha aes-192-sha aes-sha 3des-sha
    crypto dynamic-map map-dynamic 2 set pfs
    crypto dynamic-map map-dynamic 2 set transform-set aes-256-sha aes-192-sha aes-sha 3des-sha
    crypto dynamic-map map-dynamic 3 set pfs
    crypto dynamic-map map-dynamic 3 set transform-set aes-256-sha aes-192-sha aes-sha 3des-sha
    crypto dynamic-map map-dynamic 4 set transform-set aes-256-sha aes-192-sha aes-sha 3des-sha
    crypto map outside_map 1 match address outside_1_cryptomap
    crypto map outside_map 1 set peer IPNOTWHATIWASEXPECTING3
    crypto map outside_map 1 set transform-set ESP-DES-SHA
    crypto map outside_map 2 match address acl-amzn
    crypto map outside_map 2 set pfs
    crypto map outside_map 2 set peer IPNOTWHATIWASEXPECTING IPNOTWHATIWASEXPECTING
    crypto map outside_map 2 set transform-set transform-amzn
    crypto map outside_map 255 ipsec-isakmp dynamic map-dynamic
    crypto map outside_map interface outside
    crypto isakmp identity address
    crypto isakmp enable outside
    crypto isakmp policy 1
    authentication pre-share
    encryption aes-256
    hash sha
    group 5
    lifetime 86400
    crypto isakmp policy 2
    authentication pre-share
    encryption aes-256
    hash sha
    group 2
    lifetime 86400
    crypto isakmp policy 3
    authentication pre-share
    encryption aes-256
    hash sha
    group 1
    lifetime 86400
    crypto isakmp policy 11
    authentication pre-share
    encryption aes-192
    hash sha
    group 5
    lifetime 86400
    crypto isakmp policy 12
    authentication pre-share
    encryption aes-192
    hash sha
    group 2
    lifetime 86400
    crypto isakmp policy 13
    authentication pre-share
    encryption aes-192
    hash sha
    group 1
    lifetime 86400
    crypto isakmp policy 21
    authentication pre-share
    encryption aes
    hash sha
    group 5
    lifetime 86400
    crypto isakmp policy 22
    authentication pre-share
    encryption aes
    hash sha
    group 2
    lifetime 86400
    crypto isakmp policy 23
    authentication pre-share
    encryption aes
    hash sha
    group 1
    lifetime 86400
    crypto isakmp policy 31
    authentication pre-share
    encryption 3des
    hash sha
    group 5
    lifetime 86400
    crypto isakmp policy 32
    authentication rsa-sig
    encryption des
    hash sha
    group 1
    lifetime 86400
    crypto isakmp policy 33
    authentication pre-share
    encryption 3des
    hash sha
    group 1
    lifetime 86400
    crypto isakmp policy 34
    authentication pre-share
    encryption 3des
    hash sha
    group 2
    lifetime 86400
    Result of the command: "show run group-policy"
    group-policy evertest internal
    group-policy evertest attributes
    dns-server value 10.100.25.252
    vpn-idle-timeout 720
    vpn-tunnel-protocol IPSec l2tp-ipsec
    pfs enable
    split-tunnel-policy tunnelspecified
    split-tunnel-network-list value vpnsplittunnel
    default-domain value domain-notcorrect.local
    group-policy petero internal
    group-policy petero attributes
    dns-server value 10.100.25.252
    vpn-idle-timeout 720
    pfs enable
    split-tunnel-policy tunnelspecified
    split-tunnel-network-list value vpnsplittunnel
    default-domain value domain-notcorrect.local
    group-policy awsfilter internal
    group-policy awsfilter attributes
    vpn-filter value amzn-filter
    group-policy vpnpptp internal
    group-policy vpnpptp attributes
    dns-server value 10.100.25.252
    vpn-tunnel-protocol l2tp-ipsec
    group-policy vanheelm internal
    group-policy vanheelm attributes
    dns-server value 10.100.25.252
    vpn-idle-timeout 720
    vpn-tunnel-protocol IPSec l2tp-ipsec
    pfs enable
    split-tunnel-policy tunnelspecified
    split-tunnel-network-list value vpnsplittunnel
    default-domain value domain-notcorrect.local
    group-policy ciscoVPNuser internal
    group-policy ciscoVPNuser attributes
    dns-server value 10.100.25.10
    vpn-idle-timeout 720
    pfs enable
    split-tunnel-policy tunnelspecified
    split-tunnel-network-list value vpnsplittunnel
    default-domain value domain-notcorrect.local
    group-policy chauhanv2 internal
    group-policy chauhanv2 attributes
    dns-server value 10.100.25.252
    vpn-idle-timeout 720
    pfs enable
    split-tunnel-policy tunnelspecified
    split-tunnel-network-list value vpnsplittunnel
    default-domain value domain-notcorrect.local
    group-policy oterop internal
    group-policy oterop attributes
    dns-server value 10.100.25.252
    vpn-idle-timeout 720
    vpn-tunnel-protocol IPSec l2tp-ipsec
    pfs enable
    split-tunnel-policy tunnelspecified
    split-tunnel-network-list value vpnsplittunnel
    default-domain value domain-notcorrect.local
    group-policy Oterop internal
    group-policy Oterop attributes
    dns-server value 10.100.25.252
    vpn-idle-timeout 30
    group-policy chauhanv internal
    group-policy chauhanv attributes
    dns-server value 10.100.25.252
    vpn-idle-timeout 30
    vpn-tunnel-protocol IPSec l2tp-ipsec
    group-policy bnixon2 internal
    group-policy bnixon2 attributes
    dns-server value 10.100.25.252
    vpn-idle-timeout 720
    vpn-tunnel-protocol IPSec l2tp-ipsec
    pfs enable
    split-tunnel-policy tunnelspecified
    split-tunnel-network-list value vpnsplittunnel
    default-domain value domain-notcorrect.local
    Result of the command: "show run tunnel-group"
    tunnel-group ciscoVPNuser type remote-access
    tunnel-group ciscoVPNuser general-attributes
    address-pool vpnippool
    default-group-policy ciscoVPNuser
    tunnel-group ciscoVPNuser ipsec-attributes
    pre-shared-key *****
    tunnel-group petero type remote-access
    tunnel-group petero general-attributes
    address-pool vpnippool
    default-group-policy petero
    tunnel-group petero ipsec-attributes
    pre-shared-key *****
    tunnel-group oterop type remote-access
    tunnel-group oterop general-attributes
    address-pool vpnippool
    default-group-policy oterop
    tunnel-group oterop ipsec-attributes
    pre-shared-key *****
    tunnel-group vanheelm type remote-access
    tunnel-group vanheelm general-attributes
    address-pool vpnippool
    default-group-policy vanheelm
    tunnel-group vanheelm ipsec-attributes
    pre-shared-key *****
    tunnel-group chauhanv type remote-access
    tunnel-group chauhanv general-attributes
    default-group-policy chauhanv
    tunnel-group Oterop type remote-access
    tunnel-group Oterop general-attributes
    default-group-policy Oterop
    tunnel-group chauhanv2 type remote-access
    tunnel-group chauhanv2 general-attributes
    address-pool vpnippool
    default-group-policy chauhanv2
    tunnel-group chauhanv2 ipsec-attributes
    pre-shared-key *****
    tunnel-group bnixon2 type remote-access
    tunnel-group bnixon2 general-attributes
    address-pool vpnippool
    default-group-policy bnixon2
    tunnel-group bnixon2 ipsec-attributes
    pre-shared-key *****
    tunnel-group vpnpptp type remote-access
    tunnel-group vpnpptp general-attributes
    address-pool vpnippool
    default-group-policy vpnpptp
    tunnel-group IPNOTWHATIWASEXPECTING4 type ipsec-l2l
    tunnel-group IPNOTWHATIWASEXPECTING4 ipsec-attributes
    pre-shared-key *****
    tunnel-group evertest type remote-access
    tunnel-group evertest general-attributes
    address-pool vpnippool
    default-group-policy evertest
    tunnel-group evertest ipsec-attributes
    pre-shared-key *****
    tunnel-group evertest ppp-attributes
    authentication ms-chap-v2
    tunnel-group IPNOTWHATIWASEXPECTING3 type ipsec-l2l
    tunnel-group IPNOTWHATIWASEXPECTING3 ipsec-attributes
    pre-shared-key *****
    tunnel-group IPNOTWHATIWASEXPECTING2 type ipsec-l2l
    tunnel-group IPNOTWHATIWASEXPECTING2 general-attributes
    default-group-policy awsfilter
    tunnel-group IPNOTWHATIWASEXPECTING2 ipsec-attributes
    pre-shared-key *****
    isakmp keepalive threshold 10 retry 3
    tunnel-group IPNOTWHATIWASEXPECTING type ipsec-l2l
    tunnel-group IPNOTWHATIWASEXPECTING general-attributes
    default-group-policy awsfilter
    tunnel-group IPNOTWHATIWASEXPECTING ipsec-attributes
    pre-shared-key *****
    isakmp keepalive threshold 10 retry 3
    Result of the command: "show vpn-sessiondb detail remote filter protocol L2TPOverIPsec"
    INFO: There are presently no active sessions of the type specified
    Result of the command: "show vpn-sessiondb detail remote filter protocol L2TPOverIPsecOverNAT"
    INFO: There are presently no active sessions of the type specified

  • Problem with nat / access rule for webserver in inside network asa 5505 7.2

    Hello,
    i have trouble setting up nat and access rule for webserver located in inside network.
    I have asa 5505 version 7.2 and it has to active interfaces, inside 192.168.123.0 and outside x.x.x.213
    Webserver has ip 192.168.123.11 and it needs to be accessed from outside, ip x.x.x.213.
    I have created an static nat rule with pat (as an appendix) and access rules from outside network to inside interface ip 192.168.123.11 (tcp 80) but no luck.
    What am i doing wrong?

    Command:
    packet-tracer input outside tcp 188.x.x.213 www 192.168.123.11 www detailed
    Phase: 1
    Type: FLOW-LOOKUP
    Subtype:
    Result: ALLOW
    Config:
    Additional Information:
    Found no matching flow, creating a new flow
    Phase: 2
    Type: ROUTE-LOOKUP
    Subtype: input
    Result: ALLOW
    Config:
    Additional Information:
    in   192.168.123.0   255.255.255.0   inside
    Phase: 3
    Type: ACCESS-LIST
    Subtype:
    Result: DROP
    Config:
    Implicit Rule
    Additional Information:
    Forward Flow based lookup yields rule:
    in  id=0x35418d8, priority=500, domain=permit, deny=true
        hits=1, user_data=0x6, cs_id=0x0, reverse, flags=0x0, protocol=0
        src ip=188.x.x.213, mask=255.255.255.255, port=0
        dst ip=0.0.0.0, mask=0.0.0.0, port=0
    Result:
    input-interface: outside
    input-status: up
    input-line-status: up
    output-interface: inside
    output-status: up
    output-line-status: up
    Action: drop
    Drop-reason: (acl-drop) Flow is denied by configured rule

  • Cisco ASA 5505 Firewall Not Allowing Incoming Traffic

    Hello,
    I am wondering if there is a very friendly cisco guru out there who can help me out.  I am trying to switch out a cisco pix 501 firewall with a cisco ASA 5505 firewall.  I am not very familiar with all of the commands for the firewalls and have always relied on a standard command line script that I use when building a new one.  Unfortunately, my script is not working with the 5505.  Can someone please let me know what I am doing wrong with the following script?  I've masked public IP info with xxx.xxx.xxx and I run it right after restoring the firewall to the factory defaults.  I am able to get out to the internet if I browse directly from one of the servers, but cannot access a web page when trying to browse to it from an outside network.
    access-list 100 permit icmp any any echo-reply
    access-list 100 permit icmp any any time-exceeded 
    access-list 100 permit icmp any any unreachable
    ip address outside xxx.xxx.xxx.94 255.255.255.224
    ip address inside 192.168.1.1 255.255.255.0
    global (outside) 1 xxx.xxx.xxx.106-xxx.xxx.xxx.116
    global (outside) 1 xxx.xxx.xxx.95
    nat (inside) 1 0.0.0.0 0.0.0.0 0 0
    route outside 0 0 xxx.xxx.xxx.93
    access-group 100 in interface outside
    nat (inside) 1 192.168.1.0 255.255.255.0
    nat (inside) 1 192.168.1.0 255.255.255.0 0 0
    outside 0.0.0.0 0.0.0.0 xxx.xxx.xxx.93 1 DHCP static
    static (inside,outside) xxx.xxx.xxx.95 192.168.1.95 netmask 255.255.255.255 0 0
    access-list 100 permit tcp any host xxx.xxx.xxx.95 eq www
    static (inside,outside) xxx.xxx.xxx.96 192.168.1.96 netmask 255.255.255.255 0 0
    access-list 100 permit tcp any host xxx.xxx.xxx.96 eq www

    Hey Craig,
    Based on your commands I think you were using 6.3 version on PIX and now you must be  moving to ASA ver 8.2.x.
    On 8.4 for interface defining use below mentioned example :
    int eth0/0
    ip add x.x.x.x y.y.y.y
    nameif outside
    no shut
    int eth0/1
    ip add x.x.x.x y.y.y.y
    nameif inside
    no shut
    nat (inside) 1 192.168.1.0 255.255.255.0
    global (outside) 1 xxx.xxx.xxx.106-xxx.xxx.xxx.116
    global (outside) 1 xxx.xxx.xxx.95
    access-list 100 permit icmp any any echo-reply
    access-list 100 permit icmp any any time-exceeded 
    access-list 100 permit icmp any any unreachable
    static (inside,outside) xxx.xxx.xxx.95 192.168.1.95 netmask 255.255.255.255 0 0
    access-list 100 permit tcp any host xxx.xxx.xxx.95 eq www
    static (inside,outside) xxx.xxx.xxx.96 192.168.1.96 netmask 255.255.255.255 0 0
    access-list 100 permit tcp any host xxx.xxx.xxx.96 eq www
    route outside 0 0 xxx.xxx.xxx.93
    access-group 100 in interface outside
    You can use two global statements as first statement would be used a dynamic NAT and second as PAT.
    If you're still not able to reach.Paste your entire config and version that you are using on ASA.

  • OWA not accessible after setting up vpn through ASA 5505

    I have a client that is running Win2003 Server R2 with Exchange Server 2003. OWA was setup and clients could connect to their exchange mailbox from the internet with no problems.
    We recently configured vpn on the ASA 5505 and now no-one can connect to OWA since that time.
    Here is what I have for a configuration.
    Any thoughts?

    I thought OWA uses tcp port 80 by default, or 443 if you use https. Ignore the rest if this fact is wrong. Otherwise ...
    In your config there is nothing to allow the www traffic in the access-list outside_access_in
    access-list outside_access_in extended permit tcp any interface outside eq 80
    and no static nat for this
    static (inside,outside) tcp interface 80 192.168.0.254 80 netmask 255.255.255.255
    I think these must have got deleted when you made the other changes ? Hope this helps.

  • ASA 5505 Isolated Networks with Site-to-Site VPN Access

    I'm in the process of setting up an ASA 5505 for a remote site and needed some assistance determining if what I want to do is possible as well as if I need to upgrade the license from Base to Security Plus.
    Remote Site ASA 5505 Interfaces:
    Outside (Interface 0) - Public Internet, Static IP (Connected to Sierra Wireless AirLink Gateway)
    AMI (Interface 1) (VLAN 742) - 10.40.31.129/25 
    SCADA (Interface 2) (VLAN 772) - 10.70.0.5/30 
    I need to ensure that the two internal VLANs cannot access/talk to one another and the "SCADA" network cannot access Internet, just remote subnets across a VPN tunnel.
    ASA will need to have three IPsec tunnels:
    Tunnel 1 to SCADA Firewall
    Remote Site - 10.70.0.4/30 Subnet
    Central Site - 10.101.41.0/24 Subnet
    Tunnel 2 to Corporate Firewall
    Remote Site - 10.40.31.129/25 Subnet
    Central Site - 192.168.110.0/24 and 192.168.210.0/24 Subnet
    Tunnel 3 to Partner Firewall
    Remote Site - 10.40.31.129/25 Subnet
    Partner Site Subnets
    The ASA is running 9.1(5) and ASDM 7.1(6).  
    I've attached a diagram of what the connections look like between sites.

    I reviewed your diagram attached and trying to give you as much as I can.
    other gurus, pls correct me if I am missing anything.
    if I remember correctly, with base license, you can set up vpn peers.
    interface Ethernet0/0
    nameif outside
    security-level 0
    ip address public ip, subnet mask
    int e0/1
    nameif AMI
    security-level 100
    ip add 10.40.31.129 255.255.255.128
    int e0/2
    nameif SCADA
    security-level 10
    ip add 10.70.0.5 255.255.255.252
    route outside 0.0.0.0 0.0.0.0 public IP
    tunnel-group 173.8.244.181 type ipsec-l2l
    tunnel-group 173.8.244.181 ipsec-attributes
     ikev1 pre-shared-key Pr3$h@r3DkEyScAdA
    tunnel-group 173.8.244.189 type ipsec-l2l
    tunnel-group 173.8.244.189 ipsec-attributes
     ikev1 pre-shared-key Pr3$h@r3DkEyC0Rp
    tunnel-group 148.80.252.60 type ipsec-l2l
    tunnel-group 148.80.252.60 ipsec-attributes
     ikev1 pre-shared-key Pr3$h@r3DkEypArTN3R
    crypto ikev1 enable outside -- enabling for outside interface
    crypto ikev1 policy 10
     authentication pre-share
     encryption aes
     hash sha
     group 2
     lifetime 86400
    crypto ikev1 policy 15
     authentication pre-share
     encryption aes-256
     hash sha
     group 2
     lifetime 86400
    crypto ikev1 policy 20
     authentication pre-share
     encryption 3des
     hash sha
     group 2
     lifetime 28800
    crypto ipsec ikev2 ipsec-proposal AES256
     protocol esp encryption aes-256
     protocol esp integrity sha-1 md5
    crypto ipsec ikev2 ipsec-proposal AES192
     protocol esp encryption aes-192
     protocol esp integrity sha-1 md5
    crypto ipsec ikev2 ipsec-proposal AES
     protocol esp encryption aes
     protocol esp integrity sha-1 md5
    crypto ipsec ikev2 ipsec-proposal 3DES
     protocol esp encryption 3des
     protocol esp integrity sha-1 md5
    crypto ipsec ikev2 ipsec-proposal DES
     protocol esp encryption des
     protocol esp integrity sha-1 md5
    crypto ipsec security-association lifetime seconds 86400
    crypto ipsec ikev1 transform-set kerseyami esp-aes-256 esp-sha-hmac
    crypto map VPN 10 match address SCADA
    crypto map VPN 10 set peer  173.8.244.181
    crypto map VPN 10 set ikev1 transform-set kerseyami
    crypto map VPN 10 set security-association lifetime seconds 86400
    crypto map VPN 20 match address CORP
    crypto map VPN 20 set peer  173.8.244.189
    crypto map VPN 20 set ikev1 transform-set kerseyami
    crypto map VPN 20 set security-association lifetime seconds 86400
    crypto map VPN 30 match address PARTNER-FW
    crypto map VPN 30 set peer 148.80.252.60   
    crypto map VPN 30 set ikev1 transform-set kerseyami
    crypto map VPN 30 set security-association lifetime seconds 86400
    access-list SCADA extended permit ip 10.40.31.128 255.255.255.128 10.101.41.0 255.255.255.0
    access-list CORP extended permit ip 10.40.31.128 255.255.255.128 192.168.110.0 255.255.255.0
    access-list PARTNER-FW extended permit ip 10.40.31.128 255.255.255.128 subnets behind your Partner-FW
    Note: on the other side of the firewalls, like SCADA side, CORP Side and Partner FW side, you need to configure same pre-shared key, same crypto ike 1 and 2 policies & same interesting traffic in order to have this working.
    let us know how this works.
    JD...

  • How can I map SSH from an outside network range to an internal host (ASA 5505)

    Cisco Adaptive Security Appliance Software Version 7.2(4)
    Device Manager Version 5.2(4)
    - External network range that needs SSH access: 8.8.8.0/24
    - Outside interface: 10.1.10.2 (NAT'd from 7.7.7.7)
    - Inside Network: 192.168.100.0/24
    - Inside host to redirect external SSH to: 192.168.100.98
    Hi All,
    I have a Cisco ASA 5505 (version above) and I have someone that needs to SSH into a box behind the ASA. I'm having a few issues trying to configure this access-list and NAT. I've tried many combinations and clearly my IOS is not as good as I thought.
    Can anyone help with this? What commands should I enter to accomplish mapping SSH from an outside network range to an internal host?
    Many thanks,
    Tarran

    This may or may not work depending on how your modem handles the natting. On your firewall try this -
    static (inside,outside) tcp interface 22 192.168.100.98 22
    then add this to your acl on the outside interface of your ASA -
    access-list outside_in permit tcp 8.8.8.0 255.255.255.0 host 10.1.10.2 eq 22
    if you don't have an acl applied then add this extra step -
    access-group outside_in in interface outside
    Jon

  • ASA 5505 VPN can't access connected network

    I have an ASA 5505 with ipsec VPN configured on it.  I am able to  connect to the ASA but I can't ping a connected network.  I get a dhcp  assigned address in the network I am trying to reach but can't access  that network on Vlan5.  Please help.
    I attached the config.

    I think final questions, can you have two nat statements that point to the same acl ie.
    access-list no_nat extended permit ip 192.168.9.0 255.255.255.0 192.168.0.0 255.255.255.0
    access-list no_nat extended permit ip 192.168.9.0 255.255.255.0 172.31.1.0 255.255.255.0
    access-list no_nat extended permit ip 192.168.5.0 255.255.255.0 192.168.5.0 255.255.255.0
    nat (inside) 0 access-list no_nat
    nat (inside) 1 192.168.9.0 255.255.255.0
    nat (fw-civic) 0 access-list no_nat
    nat (fw-civic) 1 192.168.5.0 255.255.255.0
    Or do I need to create a new acl for the fw-civic interface?
    Thanks

  • Cisco ASA 5505 - 2 internal Networks

    Hi new to ASA's,
    Been trying to get the following setup working for ages but can't see what I am missing:
    (Got image from another post but exactly what I want but cannot get working)
    I can get ping between subnets but nothing else and Lan 2 cannot get to internet.
    The reolution for this guy was the following I believe; (from his config he has ASA v8.2)
    same-security-traffic permit intra-interface
    access-list NONAT permit ip 192.168.50.0 255.255.255.0 10.0.50.0255.255.255.0
    access-list NONAT permit ip 10.0.50.0 255.255.255.0 192.168.50.0 255.255.255.0
    nat (inside) 0 access-list NONAT
    I have tried this but I have ASA v8.4 and whilst commands 1 - 3 work command 4 doesn't.
    I get a message about the command being deprecated. I couldn't find a new version I could understand.
    Hope nothing stupid and simple but any help greatly appreciated.
    BTW, I have reset my ASA back to defaults except internet access is working and internet LAN as I made some many changes I feared one my conflict with the other.
    Many thanks for any views or help.

    Hi Jumora,
    Thanks for the reply.
    The 192 network behind the ASA can access the internet but the 10 network past the 1841 router can't.
    I have setup tcp bypass already as that got me at least remote access to the PC's on the 10 network from the 192 network.
    I had the 1841 router set to use the interface on the 192 subnet as the route to the 0.0.0.0 0.0.0.0 network but I couldn't get out but have just changed this to go to the inside interface of the ASA and can now ping 8.8.8.8 for example but still not internet access.
    Also I have found that the ASA seems to occasionally when it feels like it block pings from the 10 subnet to devices in the 192 subnet...... annoying for testing! but I can still access shares even though the ping fails.
    e.g. as per above yesterday it stopped when I enabled icmp error inspection but when I switched that off it worked again. Then suddenly again today with no changes it has stopped working again, drives me nuts the inconsistency!
    I couldn't find an attach option for the show tech so it has made this post massive.... apologies for that....
    ASA5505# show tech
    Cisco Adaptive Security Appliance Software Version 8.4(4)1
    Device Manager Version 6.4(9)
    Compiled on Thu 14-Jun-12 11:20 by builders
    System image file is "disk0:/asa844-1-k8.bin"
    Config file at boot was "startup-config"
    ASA5505 up 8 days 23 hours
    Hardware:   ASA5505, 512 MB RAM, CPU Geode 500 MHz
    Internal ATA Compact Flash, 128MB
    BIOS Flash M50FW016 @ 0xfff00000, 2048KB
    Encryption hardware device : Cisco ASA-5505 on-board accelerator (revision 0x0)
                                 Boot microcode   : CN1000-MC-BOOT-2.00
                                 SSL/IKE microcode: CNLite-MC-SSLm-PLUS-2.03
                                 IPSec microcode  : CNlite-MC-IPSECm-MAIN-2.06
                                 Number of accelerators: 1
    0: Int: Internal-Data0/0    : address is 4403.a7a2.e7c7, irq 11
    1: Ext: Ethernet0/0         : address is 4403.a7a2.e7bf, irq 255
    2: Ext: Ethernet0/1         : address is 4403.a7a2.e7c0, irq 255
    3: Ext: Ethernet0/2         : address is 4403.a7a2.e7c1, irq 255
    4: Ext: Ethernet0/3         : address is 4403.a7a2.e7c2, irq 255
    5: Ext: Ethernet0/4         : address is 4403.a7a2.e7c3, irq 255
    6: Ext: Ethernet0/5         : address is 4403.a7a2.e7c4, irq 255
    7: Ext: Ethernet0/6         : address is 4403.a7a2.e7c5, irq 255
    8: Ext: Ethernet0/7         : address is 4403.a7a2.e7c6, irq 255
    9: Int: Internal-Data0/1    : address is 0000.0003.0002, irq 255
    10: Int: Not used            : irq 255
    11: Int: Not used            : irq 255
    Licensed features for this platform:
    Maximum Physical Interfaces       : 8              perpetual
    VLANs                             : 3              DMZ Restricted
    Dual ISPs                         : Disabled       perpetual
    VLAN Trunk Ports                  : 0              perpetual
    Inside Hosts                      : 50             perpetual
    Failover                          : Disabled       perpetual
    VPN-DES                           : Enabled        perpetual
    VPN-3DES-AES                      : Enabled        perpetual
    AnyConnect Premium Peers          : 2              perpetual
    AnyConnect Essentials             : Disabled       perpetual
    Other VPN Peers                   : 10             perpetual
    Total VPN Peers                   : 12             perpetual
    Shared License                    : Disabled       perpetual
    AnyConnect for Mobile             : Disabled       perpetual
    AnyConnect for Cisco VPN Phone    : Disabled       perpetual
    Advanced Endpoint Assessment      : Disabled       perpetual
    UC Phone Proxy Sessions           : 2              perpetual
    Total UC Proxy Sessions           : 2              perpetual
    Botnet Traffic Filter             : Disabled       perpetual
    Intercompany Media Engine         : Disabled       perpetual
    This platform has a Base license.
    Serial Number: JMX3434343T
    Running Permanent Activation Key: 0x8509ef7f 0x2cff5895 0xa4675895 0x7989798 0xc1323132
    Configuration register is 0x1
    Configuration last modified by enable_15 at 16:21:28.863 UTC Wed Oct 23 2013
    ------------------ show disk0: controller ------------------
    Flash Model: SMART CF
    ------------------ show clock ------------------
    04:43:59.822 UTC Thu Oct 24 2013
    ------------------ show crashinfo ------------------
    No crash file found.
    ------------------ show module ------------------
    Mod Card Type                                    Model              Serial No.
      0 ASA 5505 Adaptive Security Appliance         ASA5505            JMX3434343T
    Mod MAC Address Range                 Hw Version   Fw Version   Sw Version    
      0 1255.a3a4.e3bf to 1233.a4a4.e4c4  0.1          1.0(12)13    8.4(4)1
    Mod SSC Application Name           Status           SSC Application Version
    Mod Status             Data Plane Status     Compatibility
      0 Up Sys             Not Applicable        
    ------------------ show memory ------------------
    Free memory:         283382600 bytes (53%)
    Used memory:         253488312 bytes (47%)
    Total memory:        536870912 bytes (100%)
    ------------------ show conn count ------------------
    76 in use, 704 most used
    ------------------ show xlate count ------------------
    80 in use, 814 most used
    ------------------ show vpn-sessiondb summary ------------------
    No sessions to display.
    ------------------ show blocks ------------------
      SIZE    MAX    LOW    CNT
         0    400    399    400
    4    100     99     99
        80    347    332    347
       256    200    192    195
      1550   6374   6306   6371
      2048   1200   1199   1200
      2560    264    264    264
      4096    100     99    100
      8192    100     99    100
    16384    100     99    100
    65536     16     15     16
    CORE  LIMIT  ALLOC   HIGH    CNT       FAILED
       0  24576     26     26     25            0
    ------------------ show blocks queue history detail ------------------
    History buffer memory usage: 2832 bytes (default)
    History analysis time limit: 100 msec
    Please see 'show blocks exhaustion snapshot' for more information
    ------------------ show interface ------------------
    Interface Ethernet0/0 "", is up, line protocol is up
      Hardware is 88E6095, BW 100 Mbps, DLY 100 usec
    Auto-Duplex(Full-duplex), Auto-Speed(100 Mbps)
    Input flow control is unsupported, output flow control is unsupported
    Available but not configured via nameif
    MAC address 4403.a7a2.e7bf, MTU not set
    IP address unassigned
    8257648 packets input, 9051289473 bytes, 0 no buffer
    Received 0 broadcasts, 0 runts, 0 giants
    0 input errors, 0 CRC, 0 frame, 0 overrun, 0 ignored, 0 abort
    0 pause input, 0 resume input
    0 L2 decode drops
    6222 switch ingress policy drops
    6399241 packets output, 1011134108 bytes, 0 underruns
    0 pause output, 0 resume output
    0 output errors, 0 collisions, 0 interface resets
    0 late collisions, 0 deferred
    0 rate limit drops
    0 switch egress policy drops
    0 input reset drops, 0 output reset drops
      Control Point Interface States:
    Interface number is 3
    Interface config status is active
    Interface state is active
    Interface Ethernet0/1 "", is up, line protocol is up
      Hardware is 88E6095, BW 100 Mbps, DLY 100 usec
    Auto-Duplex(Full-duplex), Auto-Speed(100 Mbps)
    Input flow control is unsupported, output flow control is unsupported
    Available but not configured via nameif
    MAC address 4403.a7a2.e7c0, MTU not set
    IP address unassigned
    1330699 packets input, 312264395 bytes, 0 no buffer
    Received 63097 broadcasts, 0 runts, 0 giants
    0 input errors, 0 CRC, 0 frame, 0 overrun, 0 ignored, 0 abort
    0 pause input, 0 resume input
    0 L2 decode drops
    0 switch ingress policy drops
    1738131 packets output, 637935280 bytes, 0 underruns
    0 pause output, 0 resume output
    0 output errors, 0 collisions, 0 interface resets
    0 late collisions, 0 deferred
    0 rate limit drops
    0 switch egress policy drops
    0 input reset drops, 0 output reset drops
      Control Point Interface States:
    Interface number is 4
    Interface config status is active
    Interface state is active
    Interface Ethernet0/2 "", is up, line protocol is up
      Hardware is 88E6095, BW 100 Mbps, DLY 100 usec
    Auto-Duplex(Full-duplex), Auto-Speed(100 Mbps)
    Input flow control is unsupported, output flow control is unsupported
    Available but not configured via nameif
    MAC address 4403.a7a2.e7c1, MTU not set
    IP address unassigned
    5028958 packets input, 693527818 bytes, 0 no buffer
    Received 28835 broadcasts, 0 runts, 0 giants
    0 input errors, 0 CRC, 0 frame, 0 overrun, 0 ignored, 0 abort
    0 pause input, 0 resume input
    0 L2 decode drops
    1 switch ingress policy drops
    7782140 packets output, 8316018900 bytes, 0 underruns
    0 pause output, 0 resume output
    0 output errors, 0 collisions, 0 interface resets
    0 late collisions, 0 deferred
    0 rate limit drops
    0 switch egress policy drops
    0 input reset drops, 0 output reset drops
      Control Point Interface States:
    Interface number is 5
    Interface config status is active
    Interface state is active
    Interface Ethernet0/3 "", is up, line protocol is up
      Hardware is 88E6095, BW 100 Mbps, DLY 100 usec
    Auto-Duplex(Full-duplex), Auto-Speed(100 Mbps)
    Input flow control is unsupported, output flow control is unsupported
    Available but not configured via nameif
    MAC address 4403.a7a2.e7c2, MTU not set
    IP address unassigned
    17048409 packets input, 21350059442 bytes, 0 no buffer
    Received 75081 broadcasts, 0 runts, 0 giants
    0 input errors, 0 CRC, 0 frame, 0 overrun, 0 ignored, 0 abort
    0 pause input, 0 resume input
    0 L2 decode drops
    18 switch ingress policy drops
    8319277 packets output, 5138543287 bytes, 0 underruns
    0 pause output, 0 resume output
    0 output errors, 0 collisions, 0 interface resets
    0 late collisions, 0 deferred
    0 rate limit drops
    0 switch egress policy drops
    0 input reset drops, 0 output reset drops
      Control Point Interface States:
    Interface number is 6
    Interface config status is active
    Interface state is active
    Interface Ethernet0/4 "", is down, line protocol is down
      Hardware is 88E6095, BW 100 Mbps, DLY 100 usec
    Auto-Duplex, Auto-Speed
    Input flow control is unsupported, output flow control is unsupported
    Available but not configured via nameif
    MAC address 4403.a7a2.e7c3, MTU not set
    IP address unassigned
    0 packets input, 0 bytes, 0 no buffer
    Received 0 broadcasts, 0 runts, 0 giants
    0 input errors, 0 CRC, 0 frame, 0 overrun, 0 ignored, 0 abort
    0 pause input, 0 resume input
    0 L2 decode drops
    0 switch ingress policy drops
    0 packets output, 0 bytes, 0 underruns
    0 pause output, 0 resume output
    0 output errors, 0 collisions, 0 interface resets
    0 late collisions, 0 deferred
    0 rate limit drops
    0 switch egress policy drops
    0 input reset drops, 0 output reset drops
      Control Point Interface States:
    Interface number is 7
    Interface config status is not active
    Interface state is active
    Interface Ethernet0/5 "", is down, line protocol is down
      Hardware is 88E6095, BW 100 Mbps, DLY 100 usec
    Auto-Duplex, Auto-Speed
    Input flow control is unsupported, output flow control is unsupported
    Available but not configured via nameif
    MAC address 4403.a7a2.e7c4, MTU not set
    IP address unassigned
    0 packets input, 0 bytes, 0 no buffer
    Received 0 broadcasts, 0 runts, 0 giants
    0 input errors, 0 CRC, 0 frame, 0 overrun, 0 ignored, 0 abort
    0 pause input, 0 resume input
    0 L2 decode drops
    0 switch ingress policy drops
    0 packets output, 0 bytes, 0 underruns
    0 pause output, 0 resume output
    0 output errors, 0 collisions, 0 interface resets
    0 late collisions, 0 deferred
    0 rate limit drops
    0 switch egress policy drops
    0 input reset drops, 0 output reset drops
      Control Point Interface States:
    Interface number is 8
    Interface config status is not active
    Interface state is active
    Interface Ethernet0/6 "", is down, line protocol is down
      Hardware is 88E6095, BW 100 Mbps, DLY 100 usec
    Auto-Duplex, Auto-Speed
    Input flow control is unsupported, output flow control is unsupported
    Available but not configured via nameif
    MAC address 4403.a7a2.e7c5, MTU not set
    IP address unassigned
    0 packets input, 0 bytes, 0 no buffer
    Received 0 broadcasts, 0 runts, 0 giants
    0 input errors, 0 CRC, 0 frame, 0 overrun, 0 ignored, 0 abort
    0 pause input, 0 resume input
    0 L2 decode drops
    0 switch ingress policy drops
    0 packets output, 0 bytes, 0 underruns
    0 pause output, 0 resume output
    0 output errors, 0 collisions, 0 interface resets
    0 late collisions, 0 deferred
    0 rate limit drops
    0 switch egress policy drops
    0 input reset drops, 0 output reset drops
      Control Point Interface States:
    Interface number is 9
    Interface config status is not active
    Interface state is active
    Interface Ethernet0/7 "", is up, line protocol is up
      Hardware is 88E6095, BW 100 Mbps, DLY 100 usec
    Auto-Duplex(Full-duplex), Auto-Speed(100 Mbps)
    Input flow control is unsupported, output flow control is unsupported
    Available but not configured via nameif
    MAC address 4403.a7a2.e7c6, MTU not set
    IP address unassigned
    7293552 packets input, 4521902362 bytes, 0 no buffer
    Received 6520 broadcasts, 0 runts, 0 giants
    0 input errors, 0 CRC, 0 frame, 0 overrun, 0 ignored, 0 abort
    0 pause input, 0 resume input
    0 L2 decode drops
    0 switch ingress policy drops
    16232858 packets output, 21234947011 bytes, 0 underruns
    0 pause output, 0 resume output
    0 output errors, 0 collisions, 0 interface resets
    0 late collisions, 0 deferred
    0 rate limit drops
    0 switch egress policy drops
    0 input reset drops, 0 output reset drops
      Control Point Interface States:
    Interface number is 10
    Interface config status is active
    Interface state is active
    Interface Internal-Data0/0 "", is up, line protocol is up
      Hardware is y88acs06, BW 1000 Mbps, DLY 10 usec
    (Full-duplex), (1000 Mbps)
    Input flow control is unsupported, output flow control is unsupported
    MAC address 4403.a2a2.e2c2, MTU not set
    IP address unassigned
    15222257 packets input, 10134321711 bytes, 0 no buffer
    Received 173531 broadcasts, 0 runts, 0 giants
    0 input errors, 0 CRC, 0 frame, 0 overrun, 0 ignored, 0 abort
    0 pause input, 0 resume input
    0 L2 decode drops, 0 demux drops
    15128507 packets output, 10256870512 bytes, 0 underruns
    0 pause output, 0 resume output
    0 output errors, 0 collisions, 0 interface resets
    0 late collisions, 0 deferred
    0 input reset drops, 0 output reset drops, 0 tx hangs
    input queue (blocks free curr/low): hardware (512/487)
    output queue (blocks free curr/low): hardware (512/450)
      Control Point Interface States:
    Interface number is 2
    Interface config status is active
    Interface state is active
    Interface Internal-Data0/1 "", is up, line protocol is up
      Hardware is 88E6095, BW 1000 Mbps, DLY 10 usec
    (Full-duplex), (1000 Mbps)
    Input flow control is unsupported, output flow control is unsupported
    MAC address 0000.0003.0002, MTU not set
    IP address unassigned
    15128465 packets input, 10256855882 bytes, 0 no buffer
    Received 1967 broadcasts, 0 runts, 0 giants
    0 input errors, 0 CRC, 0 frame, 0 overrun, 0 ignored, 0 abort
    0 pause input, 0 resume input
    0 switch ingress policy drops
    15222217 packets output, 10134318430 bytes, 0 underruns
    0 pause output, 0 resume output
    0 output errors, 0 collisions, 0 interface resets
    0 late collisions, 0 deferred
    0 switch egress policy drops
    0 input reset drops, 0 output reset drops
      Control Point Interface States:
    Interface number is 11
    Interface config status is active
    Interface state is active
    Interface Vlan1 "inside", is up, line protocol is up
      Hardware is EtherSVI, BW 100 Mbps, DLY 100 usec
    MAC address 4403.a7a2.e7c7, MTU 1500
    IP address 192.168.1.1, subnet mask 255.255.255.0
      Traffic Statistics for "inside":
    4183727 packets input, 523675346 bytes
    5702790 packets output, 5851485425 bytes
    142576 packets dropped
          1 minute input rate 22 pkts/sec,  2839 bytes/sec
          1 minute output rate 30 pkts/sec,  22751 bytes/sec
          1 minute drop rate, 0 pkts/sec
          5 minute input rate 33 pkts/sec,  3746 bytes/sec
          5 minute output rate 46 pkts/sec,  20906 bytes/sec
          5 minute drop rate, 1 pkts/sec
      Control Point Interface States:
    Interface number is 14
    Interface config status is active
    Interface state is active
    Interface Vlan2 "outside", is up, line protocol is up
      Hardware is EtherSVI, BW 100 Mbps, DLY 100 usec
    MAC address 4403.a7a2.e7c7, MTU 1492
    IP address 98.22.77.33, subnet mask 255.255.255.255
      Traffic Statistics for "outside":
    10541983 packets input, 11433817622 bytes
    3793777 packets output, 526586888 bytes
    13654 packets dropped
          1 minute input rate 47 pkts/sec,  41657 bytes/sec
          1 minute output rate 18 pkts/sec,  2802 bytes/sec
          1 minute drop rate, 0 pkts/sec
          5 minute input rate 80 pkts/sec,  38519 bytes/sec
          5 minute output rate 29 pkts/sec,  3749 bytes/sec
          5 minute drop rate, 0 pkts/sec
      Control Point Interface States:
    Interface number is 15
    Interface config status is active
    Interface state is active
    Interface Virtual0 "_internal_loopback", is up, line protocol is up
      Hardware is Virtual          MAC address 0000.0000.0000, MTU 1500
    IP address 127.0.0.1, subnet mask 255.255.255.0
      Traffic Statistics for "_internal_loopback":
    1 packets input, 28 bytes
    1 packets output, 28 bytes
    1 packets dropped
          1 minute input rate 0 pkts/sec,  0 bytes/sec
          1 minute output rate 0 pkts/sec,  0 bytes/sec
          1 minute drop rate, 0 pkts/sec
          5 minute input rate 0 pkts/sec,  0 bytes/sec
          5 minute output rate 0 pkts/sec,  0 bytes/sec
          5 minute drop rate, 0 pkts/sec
      Control Point Interface States:
    Interface number is 12
    Interface config status is active
    Interface state is active
    ------------------ show cpu usage ------------------
    CPU utilization for 5 seconds = 12%; 1 minute: 8%; 5 minutes: 8%
    ------------------ show cpu hogging process ------------------
    Process:      Unicorn Admin Handler, PROC_PC_TOTAL: 1, MAXHOG: 23, LASTHOG: 23
    LASTHOG At:   06:01:57 UTC Oct 15 2013
    PC:           0x0853e1f4 (suspend)
    Process:      Unicorn Admin Handler, NUMHOG: 1, MAXHOG: 23, LASTHOG: 23
    LASTHOG At:   06:01:57 UTC Oct 15 2013
    PC:           0x0853e1f4 (suspend)
    Call stack:   0x0853e1f4  0x0853ec36  0x0854182c  0x0869cc4b  0x08415ae7  0x0840ae40  0x0806e6cf
                  0x08aade2b  0x0806e6cf  0x084a0a44  0x0849986d  0x08499aac  0x08499dd6  0x084a0909
    Process:      Unicorn Admin Handler, PROC_PC_TOTAL: 2, MAXHOG: 18, LASTHOG: 18
    LASTHOG At:   06:01:57 UTC Oct 15 2013
    PC:           0x0853fb48 (suspend)
    Process:      Unicorn Admin Handler, NUMHOG: 2, MAXHOG: 18, LASTHOG: 18
    LASTHOG At:   06:01:57 UTC Oct 15 2013
    PC:           0x0853fb48 (suspend)
    Call stack:   0x0853fb48  0x0853fd1d  0x0853e1bc  0x0853ec36  0x0854182c  0x0869cc4b  0x08415ae7
          0x0840ae40  0x0806e6cf  0x08aade2b  0x0806e6cf  0x084a0a44  0x0849986d  0x08499aac
    Process:      Unicorn Admin Handler, PROC_PC_TOTAL: 2, MAXHOG: 24, LASTHOG: 24
    LASTHOG At:   06:01:57 UTC Oct 15 2013
    PC:           0x084167d2 (suspend)
    Process:      Unicorn Admin Handler, NUMHOG: 2, MAXHOG: 24, LASTHOG: 24
    LASTHOG At:   06:01:57 UTC Oct 15 2013
    PC:           0x084167d2 (suspend)
    Call stack:   0x08538afd  0x0853fa3a  0x0853fd1d  0x0853e1bc  0x0853ec36  0x0854182c  0x0869cc4b
                  0x08415ae7  0x0840ae40  0x0806e6cf  0x08aade2b  0x0806e6cf  0x084a0a44  0x0849986d
    Process:      Unicorn Admin Handler, PROC_PC_TOTAL: 1, MAXHOG: 12, LASTHOG: 12
    LASTHOG At:   06:01:57 UTC Oct 15 2013
    PC:           0x08ee9b4e (suspend)
    Process:      Unicorn Admin Handler, NUMHOG: 1, MAXHOG: 12, LASTHOG: 12
    LASTHOG At:   06:01:57 UTC Oct 15 2013
    PC:           0x08ee9b4e (suspend)
    Call stack:   0x08ee9e12  0x084a1032  0x0849986d  0x08499aac  0x08499dd6  0x084a0909  0x080689bc
    Process:      Dispatch Unit, PROC_PC_TOTAL: 2, MAXHOG: 12, LASTHOG: 12
    LASTHOG At:   06:01:57 UTC Oct 15 2013
    PC:           0x081e208a (suspend)
    Process:      Dispatch Unit, NUMHOG: 2, MAXHOG: 12, LASTHOG: 12
    LASTHOG At:   06:01:57 UTC Oct 15 2013
    PC:           0x081e208a (suspend)
    Call stack:   0x081e208a  0x080689bc
    Process:      Unicorn Admin Handler, NUMHOG: 1, MAXHOG: 180, LASTHOG: 180
    LASTHOG At:   07:24:33 UTC Oct 19 2013
    PC:           0x0806a8c2 (suspend)
    Call stack:   0x0806a8c2  0x08a8ebd7  0x08a8f7c8  0x08a914fa  0x080ddd6f  0x080df9db  0x080f4132
                  0x080f5b16  0x080dd956  0x080de0ef  0x080de876  0x080dea37  0xdd6e6c1c  0xdd6e71b5
    Process:      rtcli async executor process, NUMHOG: 14, MAXHOG: 94, LASTHOG: 82
    LASTHOG At:   07:28:06 UTC Oct 19 2013
    PC:           0x08f262e3 (suspend)
    Call stack:   0x0806a881  0x08f262e3  0x08f432a2  0x09064ba8  0x0903dfa9  0x0904f88d  0x0903ed70
                  0x09036221  0x0903d29b  0x0903d49f  0x09035ffa  0x09055321  0x0903dfa9  0x0904f88d
    Process:      rtcli async executor process, PROC_PC_TOTAL: 27, MAXHOG: 319, LASTHOG: 88
    LASTHOG At:   07:28:06 UTC Oct 19 2013
    PC:           0x08f4212d (suspend)
    Process:      rtcli async executor process, NUMHOG: 27, MAXHOG: 319, LASTHOG: 88
    LASTHOG At:   07:28:06 UTC Oct 19 2013
    PC:           0x08f4212d (suspend)
    Call stack:   0x08069faa  0x08f4212d  0x08f260b6  0x08f27b85  0x08f27c35  0xcb147b98
    Process:      rtcli async executor process, PROC_PC_TOTAL: 12, MAXHOG: 45, LASTHOG: 10
    LASTHOG At:   07:28:14 UTC Oct 19 2013
    PC:           0x08f2594b (suspend)
    Process:      rtcli async executor process, NUMHOG: 12, MAXHOG: 45, LASTHOG: 10
    LASTHOG At:   07:28:14 UTC Oct 19 2013
    PC:           0x08f2594b (suspend)
    Call stack:   0x0806a881  0x08f2594b  0x08f27b85  0x08f27c35  0xcb147b98
    Process:      Unicorn Admin Handler, NUMHOG: 4, MAXHOG: 11, LASTHOG: 11
    LASTHOG At:   07:28:14 UTC Oct 19 2013
    PC:           0x0806a8c2 (suspend)
    Call stack:   0x0806a8c2  0x08a8ebd7  0x08b9aa46  0x08b9ad0e  0x080dc76f  0xdd6e6961  0xdd6e71b5
                  0xdd6e7b07  0xdd6e8d5c  0xdd6e138d  0xdd6e247a  0x080dcb22  0x0849f899  0x084981c7
    Process:      rtcli async executor process, PROC_PC_TOTAL: 83, MAXHOG: 298, LASTHOG: 119
    LASTHOG At:   07:28:16 UTC Oct 19 2013
    PC:           0x08f262e3 (suspend)
    Process:      rtcli async executor process, NUMHOG: 47, MAXHOG: 298, LASTHOG: 119
    LASTHOG At:   07:28:16 UTC Oct 19 2013
    PC:           0x08f262e3 (suspend)
    Call stack:   0x0806a881  0x08f262e3  0x08f38fad  0x08f3acc0  0x0905a29e  0x0905b2ba  0x0903dfa9
                  0x0903ecb5  0x0904f6f5  0x0903ed70  0x09036221  0x0903d29b  0x0903d49f  0x09035ffa
    Process:      Unicorn Admin Handler, NUMHOG: 3, MAXHOG: 180, LASTHOG: 180
    LASTHOG At:   07:28:16 UTC Oct 19 2013
    PC:           0x0806a8c2 (suspend)
    Call stack:   0x0806a881  0x0806a8c2  0x0816261b  0x095302a7  0x0954abef  0x0954acc3  0x0815aabe
                  0x08134da6  0x08c64632  0x08ea8079  0x08ea8481  0x08ea85f7  0x08f41adc  0x0806e6cf
    Process:      Unicorn Admin Handler, NUMHOG: 3, MAXHOG: 15, LASTHOG: 15
    LASTHOG At:   07:28:20 UTC Oct 19 2013
    PC:           0x0806a8c2 (suspend)
    Call stack:   0x0806a881  0x0806a8c2  0x0947a399  0x0946d24d  0x0946d364  0x08c2b0e6  0x08c38f65
                  0x08ea810b  0x08ea8481  0x08ea85f7  0x08f41adc  0x0806e6cf  0x08f3cc48  0x092afca6
    Process:      Unicorn Admin Handler, NUMHOG: 3, MAXHOG: 64, LASTHOG: 64
    LASTHOG At:   07:28:20 UTC Oct 19 2013
    PC:           0x0806a8c2 (suspend)
    Call stack:   0x0806a881  0x0806a8c2  0x0947a3e4  0x09479cf9  0x094750eb  0x08c3f645  0x08c3fcab
                  0x08c2b235  0x08c38f65  0x08ea810b  0x08ea8481  0x08ea85f7  0x08f41adc  0x0806e6cf
    Process:      IP Thread, NUMHOG: 4, MAXHOG: 14, LASTHOG: 14
    LASTHOG At:   07:28:24 UTC Oct 19 2013
    PC:           0x0806a8c2 (suspend)
    Call stack:   0x0806a8c2  0x0947a399  0x0946d24d  0x0946d364  0x08c2b0e6  0x08c38f65  0x08ea810b
                  0x08ea8481  0x08ea85f7  0x08ea5f86  0x090e086e  0x090e0b6e  0x090b9a99  0x090b6b00
    Process:      Unicorn Admin Handler, PROC_PC_TOTAL: 22, MAXHOG: 180, LASTHOG: 64
    LASTHOG At:   07:28:24 UTC Oct 19 2013
    PC:           0x0806a8c2 (suspend)
    Process:      IP Thread, NUMHOG: 4, MAXHOG: 64, LASTHOG: 64
    LASTHOG At:   07:28:24 UTC Oct 19 2013
    PC:           0x0806a8c2 (suspend)
    Call stack:   0x0806a8c2  0x0947a3e4  0x09479cf9  0x094750eb  0x08c3f645  0x08c3fcab  0x08c2b235
                  0x08c38f65  0x08ea810b  0x08ea8481  0x08ea85f7  0x08ea5f86  0x090e086e  0x090e0b6e
    CPU hog threshold (msec): 10.240
    Last cleared: None
    ------------------ show process ------------------
        PC       SP       STATE       Runtime    SBASE     Stack Process
    Lwe 0x08058ba4 0xc82baf84 0x0a345788          0 0xc82b7078 15760/16384 block_diag
    Mrd 0x081e1e11 0xc82ed54c 0x0a346144     430188 0xc82cd6e0 120548/131072 Dispatch Unit
    Msi 0x087509a4 0xc82fdcb4 0x0a3458b0        713 0xc82f9da8 15688/16384 WebVPN KCD Process
    Msi 0x09200c7b 0xc839b3d4 0x0a3458b0       3466 0xc83974c8 15688/16384 y88acs06 OneSec Thread
    Mwe 0x080718dd 0xc83a3804 0x0a3458b0          0 0xc839f948 15808/16384 Reload Control Thread
    Mwe 0x080849b9 0xc83ae79c 0x0a346e2c          0 0xc83aabe0 15256/16384 aaa
    Mwe 0x08f4212d 0xc8d3d1e4 0x0a3458b0          9 0xc83aed78 15056/16384 UserFromCert Thread
    Mwe 0x08f4212d 0xc9003fe4 0x0a3458b0         14 0xc83b2f50 14528/16384 aaa_shim_thread
    Mwe 0x080b477c 0xc83bfa1c 0x0a347eb4          0 0xc83bbb20 15760/16384 CMGR Server Process
    Mwe 0x080b6ded 0xc83c3b64 0x0a3458b0          0 0xc83bfcb8 15832/16384 CMGR Timer Process
    Lwe 0x081e0474 0xc83d83bc 0x0a3568e0          0 0xc83d44b0 15488/16384 dbgtrace
    Mwe 0x084de0ed 0xc83ef574 0x0a3458b0          0 0xc83e76d8 31680/32768 idfw_proc
    Mwe 0x084ea35b 0xc83f75b4 0x0a3458b0          0 0xc83ef708 32216/32768 idfw_service
    Mwe 0x084f5fc5 0xc83fb70c 0x0a3458b0          0 0xc83f78a0 15524/16384 idfw_adagent
    Mwe 0x085351b5 0xc84038dc 0x0a3458b0         89 0xc83ffbd0 11568/16384 eswilp_svi_init
    Mwe 0x08f4212d 0xc8770564 0x0a3458b0          0 0xc8433aa0 15280/16384 netfs_thread_init
    Mwe 0x09576795 0xc844c10c 0x0a3458b0          0 0xc8448290 15848/16384 Chunk Manager
    Msi 0x08ae10be 0xc84508ac 0x0a3458b0       3523 0xc844c9c0 15656/16384 PIX Garbage Collector
    Mwe 0x08ac328a 0xc8461a0c 0x0a1d5d24          0 0xc845db00 16104/16384 IP Address Assign
    Mwe 0x08d0477a 0xc85f7534 0x0a251838          0 0xc85f3628 16104/16384 QoS Support Module
    Mwe 0x08b5c32a 0xc85fb70c 0x0a1d6c88          0 0xc85f7800 16104/16384 Client Update Task
    Lwe 0x095d54f5 0xc860009c 0x0a3458b0     109750 0xc85fc1f0 14448/16384 Checkheaps
    Mwe 0x08d093ed 0xc861080c 0x0a3458b0        454 0xc86089a0 19328/32768 Quack process
    Mwe 0x08d8569d 0xc86189c4 0x0a3458b0        533 0xc8610b38 31952/32768 Session Manager
    Mwe 0x08ed964d 0xc8620cd4 0xcadf5b08          8 0xc861ce68 15464/16384 uauth
    Mwe 0x08e66621 0xc8624f0c 0x0a264a10          0 0xc8621000 15632/16384 Uauth_Proxy
    Msp 0x08ea87de 0xc86313d4 0x0a3458b0        561 0xc862d4c8 15688/16384 SSL
    Mwe 0x08ed72d4 0xc863554c 0x0a26bc14          0 0xc8631660 15708/16384 SMTP
    Mwe 0x08ed170c 0xc86396a4 0x0a26af38      23255 0xc86357f8 13608/16384 Logger
    Mwe 0x08ecfd1d 0xc863d80c 0x0a3458b0          0 0xc8639990 15784/16384  Syslog Retry Thread
    Mwe 0x08ecadf5 0xc86419d4 0x0a3458b0          0 0xc863db28 15600/16384 Thread Logger
    Mwe 0x08ed50b4 0xc866457c 0x0a26b5e0          0 0xc8660680 15464/16384 syslogd
    Mwe 0x09132032 0xc8681094 0x0a2a5688          0 0xc867d1a8 15328/16384 vpnlb_thread
    Mwe 0x092037ec 0xc86916c4 0x0a2aa9e8          0 0xc868d808 16024/16384 pci_nt_bridge
    Mwe 0x082beb95 0xc8756e44 0x0a3458b0          0 0xc8752fb8 15864/16384 TLS Proxy Inspector
    Msi 0x08da221c 0xc87d44a4 0x0a3458b0       2749 0xc87d0598 15688/16384 emweb/cifs_timer
    Mwe 0x08852cc4 0xc88291f4 0x0a1c4c44          0 0xc88252f8 15712/16384 netfs_mount_handler
    Msi 0x086b4248 0xc8316454 0x0a3458b0      27304 0xc8312568 15312/16384 arp_timer
    Mwe 0x086bc58e 0xc8447fb4 0x0a371110          0 0xc84440f8 16024/16384 arp_forward_thread
    Mwe 0x08eddb77 0xc8f2e27c 0x0a26c680          0 0xc8f2a380 15672/16384 tcp_fast
    Mwe 0x08ee69a8 0xc8f3229c 0x0a26c680          0 0xc8f2e3b0 15656/16384 tcp_slow
    Mwe 0x08f1df34 0xc8f42fac 0x0a2745d0          0 0xc8f3f0b0 16000/16384 udp_timer
    Mwe 0x0814110d 0xc8fb133c 0xc83ca8d0          4 0xc8fad4a0 15664/16384 IPsec message handler
    Mwe 0x087515c6 0xc8fdc834 0x0a376060          1 0xc8fd8958 16056/16384 Lic TMR
    Mwe 0x087513bc 0xc8fe0884 0x0a1c0ea0        242 0xc8fdc988 16088/16384 Lic HA
    Msi 0x08153267 0xc84270dc 0x0a3458b0      54986 0xc8423440 13872/16384 CTM message handler
    Mwe 0x0811bd2d 0xc843bb8c 0x0a3458b0          0 0xc8437ce0 15832/16384 CTCP Timer process
    Mwe 0x090d3d95 0xc843fbac 0x0a3458b0          0 0xc843bd10 15816/16384 L2TP data daemon
    Mwe 0x090d6605 0xc9b5b24c 0x0a3458b0          0 0xc9b573b0 15816/16384 L2TP mgmt daemon
    Mwe 0x090c2b27 0xc9b9339c 0x0a29a3ec       2228 0xc9b8f4e0 15480/16384 ppp_timer_thread
    Msi 0x0913239d 0xc9b973ec 0x0a3458b0       4093 0xc9b93510 15640/16384 vpnlb_timer_thread
    Mwe 0x081c7708 0xc9c67c84 0x0a13ef88       2899 0xc9c47f18 118548/131072 tmatch compile thread
    Mwe 0x08d38b2d 0xcac940cc 0x0a3458b0          0 0xcac90210 15848/16384 ICMP event handler
    Mwe 0x0908081d 0xcac98254 0x0a3458b0          0 0xcac943a8 15832/16384 Dynamic Filter VC Housekeeper
    Mwe 0x08a1b612 0xcacc47f4 0x0a3458b0        819 0xcacc0938 13860/16384 IP Background
    Mwe 0x08c26e63 0xcaed904c 0x0a3458b0          0 0xcaed51a0 15832/16384 Crypto CA
    Mwe 0x08c60c18 0xcaedd1e4 0x0a3458b0          0 0xcaed9338 15896/16384 CERT API
    Mwe 0x08c257d5 0xcaee6e24 0x0a3458b0          0 0xcaee2f58 15928/16384 Crypto PKI RECV
    Mwe 0x0878dd85 0xc862d1cc 0x0a3458b0        187 0xc8629330 15272/16384 ESW_MRVL switch interrupt service
    Mwe 0x08cae62c 0xc866c89c 0x0a1ea7e0          0 0xc86689b0 15832/16384 lina_int
    Mrd 0x0959948b 0xc8684f1c 0x0a346144   28493079 0xc8681340 13824/16384 esw_stats
    Lsi 0x08af3199 0xc86958bc 0x0a3458b0        152 0xc86919a0 15704/16384 uauth_urlb clean
    Lwe 0x08acbd76 0xc83ff8b4 0x0a3458b0       4432 0xc83fba38 14308/16384 pm_timer_thread
    Mwe 0x08555f8d 0xc8418b0c 0x0a3458b0          0 0xc8414c60 15832/16384 IKE Common thread
    Mwe 0x0858cecd 0xcaf8688c 0x0a3458b0          0 0xcaf82a60 15704/16384 IKE Timekeeper
    Mwe 0x0857bad1 0xcaf8ccc4 0x0a1bc678          1 0xcaf890e8 12116/16384 IKE Daemon
    Mwe 0x08629eb3 0xcaf90c64 0x0a3458b0        964 0xcaf8d118 14744/16384 IKEv2 Daemon
    Mwe 0x08628e7c 0xcaf94ff4 0x0a3458b0       1095 0xcaf91148 15640/16384 IKEv2 DPD Client Process
    Mwe 0x08e7d2e4 0xcafafd7c 0x0a2690f4          0 0xcafabe90 16072/16384 RADIUS Proxy Event Daemon
    Mwe 0x08e41f35 0xcafb3d74 0xcb07e358          7 0xcafb0028 14912/16384 RADIUS Proxy Listener
    Mwe 0x08e7ca0d 0xcafb806c 0x0a3458b0          0 0xcafb41c0 15832/16384 RADIUS Proxy Time Keeper
    Mwe 0x086a1e44 0xcafbc184 0x0a3710c8          0 0xcafb8358 15264/16384 Integrity FW Task
    Mrd 0x082c923a 0xcaffce54 0x0a346144          0 0xcaff8f98 14552/16384 CP Threat-Detection Processing
    Mwe 0x081fb74e 0xcb0cc4bc 0x09c4a8bc       2497 0xcb0acd60 122448/131072 ci/console
    Msi 0x08b0ea8c 0xcb0d0e14 0x0a3458b0     217583 0xcb0ccef8 14004/16384 update_cpu_usage
    Mwe 0x08ef5ff5 0xcb0d4ecc 0x0a3458b0         77 0xcb0d1090 15360/16384 npshim_thread
    Msi 0x08b0eb14 0xcb0e1224 0x0a3458b0          0 0xcb0dd428 13104/16384 NIC status poll
    Mwe 0x08dd5f2c 0xcb0e54bc 0x0a259ec8        228 0xcb0e15c0 15540/16384 SNMP Notify Thread
    Mwe 0x086aba0e 0xcb12ebe4 0x0a37170c     235813 0xcb126d08 25428/32768 IP Thread
    Mwe 0x086b31fe 0xcb132d9c 0x0a371100       9150 0xcb12eea0 9700/16384 ARP Thread
    Mwe 0x084be3ae 0xcb136f8c 0x0a3716c8       1743 0xcb1331b0 12696/16384 icmp_thread
    Mwe 0x08f1f443 0xcb13b1e4 0x0a3458b0        158 0xcb137348 15728/16384 udp_thread
    Mwe 0x08ee0f44 0xcb13f0bc 0x0a37178c          0 0xcb13b4e0 15288/16384 tcp_thread
    Mwe 0x08f4212d 0xcb1bccd4 0x0a3458b0      12848 0xcb13fd70 26600/32768 rtcli async executor process
    Mwe 0x090e408d 0xcb4dff64 0x0a3458b0          0 0xcb4dc0a8 14608/16384 PPPOE background daemon
    Mwe 0x090e53c4 0xcb4e3fb4 0x0a29aa4c          1 0xcb4e00d8 14656/16384 PPPOE CLI daemon
    Mwe 0x0824ff45 0xcb501e4c 0x0a3458b0        258 0xcb4fdf90 15624/16384 Timekeeper
    Mwe 0x08e41f35 0xcb89a6d4 0xcb89eb10          7 0xcb896998 15392/16384 EAPoUDP-sock
    Mwe 0x0822323d 0xcb89e544 0x0a3458b0          0 0xcb89a9c8 15016/16384 EAPoUDP
    Mwe 0x08204371 0xcb3df9dc 0x0a3458b0        149 0xcb3dbb20 15168/16384 DHCPD Timer
    Mwe 0x082066a1 0xcb3e6404 0x0a3458b0       1286 0xcb3e25a8 7172/16384 dhcp_daemon
    Mwe 0x0910dfd4 0xcbc3b4e4 0x0a2a5380          0 0xcbc335e8 32472/32768 vpnfol_thread_msg
    Msi 0x09116252 0xcbc3fac4 0x0a3458b0       2657 0xcbc3bbd8 15656/16384 vpnfol_thread_timer
    Mwe 0x09114882 0xcbc44074 0x0a2a53c0          0 0xcbc401c8 16008/16384 vpnfol_thread_sync
    Msi 0x09115fdc 0xcbc486b4 0x0a3458b0      11061 0xcbc447b8 15672/16384 vpnfol_thread_unsent
    Mwe 0x0869e365 0xc8689384 0x0a3458b0          0 0xc86854d8 15832/16384 Integrity Fw Timer Thread
    Msi 0x08852fd6 0xc868d55c 0x0a3458b0        206 0xc8689670 15656/16384 netfs_vnode_reclaim
    Mwe 0x08f4212d 0xcb2a1914 0x0a3458b0       1277 0xcbd38510 15008/16384 Unicorn Proxy Thread
    Mwe 0x0825afcb 0xcbc61254 0x0a3458b0        335 0xcbc5d788 14272/16384 emweb/https
    Mwe 0x08eef828 0xcbd4dd0c 0xcbd4fd7c          0 0xcbd49fd0 14888/16384 listen/telnet
    Mwe 0x08aac530 0xcbdbd754 0xcbd6c9fc        102 0xcbd9def8 127432/131072 Unicorn Admin Handler
    Mwe 0x08aab345 0xcbddd644 0x0a3458b0        105 0xcbdbdf28 123712/131072 Unicorn Admin Handler
    Mwe 0x08cd7c6f 0xcaf358cc 0x0a49edc8          0 0xcaf31bb0 15384/16384 qos_metric_daemon
    Mwe 0x08218c82 0xcb2693fc 0x0a3458b0          3 0xcb265560 13248/16384 DHCP Client
    Mwe 0x08f1d929 0xcb4bb0fc 0xc8f3ece4          0 0xcb4b3300 31552/32768 DHCPC Receiver
    M*  0x08a86f55 0xdcc1df2c 0x0a346144        274 0xcb34deb8 19696/32768 telnet/ci
    -           -          -          -          0          -      -      DATAPATH-0-455
    -           -          -          -  744377118          -      -      scheduler
    -           -          -          -  774156778          -      -      total elapsed
    ------------------ show kernel process ------------------
    PID PPID PRI NI      VSIZE      RSS      WCHAN STAT  RUNTIME COMMAND
      1    0  20  0    2080768      616 3725686580    S      630 init
      2    0  15 -5          0        0 3725738556    S        0 kthreadd
      3    2  15 -5          0        0 3725692956    S        0 ksoftirqd/0
      4    2  15 -5          0        0 3725728656    S        0 events/0
      5    2  15 -5          0        0 3725728656    S        0 khelper
    50    2  15 -5          0        0 3725728656    S        0 kblockd/0
    53    2  15 -5          0        0 3726777703    S        0 kseriod
    99    2  20  0          0        0 3725848262    S        0 pdflush
    100    2  20  0          0        0 3725848262    S        0 pdflush
    101    2  15 -5          0        0 3725861131    S        0 kswapd0
    102    2  15 -5          0        0 3725728656    S        0 aio/0
    103    2  15 -5          0        0 3725728656    S        0 nfsiod
    214    2  15 -5          0        0 3725728656    S        0 hid_compat
    215    2  15 -5          0        0 3725728656    S        0 rpciod/0
    240    1  16 -4    1789952      600 3725997327    S        4 udevd
    272  240  18 -2    1785856      564 3725997327    S        0 udevd
    277  240  18 -2    1785856      552 3725997327    S        0 udevd
    421    1  20  0    5201920     1600 4294967295    S       11 lwsmd
    423  421  20  0   16736256     3600 4294967295    S      102 lwregd
    448    1  20  0    2084864      512 3725686580    S        1 sh
    449  448  20  0   10186752      528 4294967295    S        2 lina_monitor
    451  449   0 -20  440270848    53000 4294967295    S 77713055 lina
    ------------------ show kernel cgroup-controller detail ------------------
    memory controller:
    memory.limit_in_bytes: unlimited
    memory.usage_in_bytes: 61665280   (11%)
    memory.max_usage_in_bytes: 64245760   (12%)
    memory.failcnt: 0
    tasks:
    group "normal"
      memory.limit_in_bytes: unlimited
      memory.usage_in_bytes: 77824   (0%)
      memory.max_usage_in_bytes: 544768   (0%)
      memory.failcnt: 0
      tasks:
           PID         RSS COMMAND                      
             1      630784 init                         
             2           0 kthreadd                     
             3           0 ksoftirqd/0                  
             4           0 events/0                     
             5           0 khelper                      
            50           0 kblockd/0                    
            53           0 kseriod                      
            99           0 pdflush                      
           100           0 pdflush                      
           101           0 kswapd0                      
           102           0 aio/0                        
           103           0 nfsiod                       
           214           0 hid_compat                   
           215           0 rpciod/0                     
           240      614400 udevd                        
           272      577536 udevd                        
           277      565248 udevd                        
           448      524288 sh                           
    group "privileged"
      memory.limit_in_bytes: unlimited
      memory.usage_in_bytes: 22327296   (4%)
      memory.max_usage_in_bytes: 22515712   (4%)
      memory.failcnt: 0
      tasks:
           PID         RSS COMMAND                      
           449      540672 lina_monitor                 
           450           0 lina_monitor                 
           451    54280192 lina                         
           452           0 lina                         
           453           0 lina                         
           454           0 lina                         
           455           0 lina                         
    group "restricted"
      memory.limit_in_bytes: 23068672   (4%)
      memory.usage_in_bytes: 1724416   (0%)
      memory.max_usage_in_bytes: 1900544   (0%)
      memory.failcnt: 0
      tasks:
           PID         RSS COMMAND                      
           421     1638400 lwsmd                        
           422           0 lwsmd                        
           423     3686400 lwregd                       
           425           0 lwregd                       
           426           0 lwregd                       
           427           0 lwregd                       
           428           0 lwregd                       
           429           0 lwregd                       
           430           0 lwsmd                        
           431           0 lwsmd                        
           432           0 lwsmd                        
           433           0 lwsmd                        
           434           0 lwsmd                        
    cpu controller:
    cpu.shares: 1024
    cpuacct.usage: 777015353084076
    tasks:
    group "normal"
      cpu.shares: 1024
      cpuacct.usage: 53525955783   (0%)
    tasks:
           PID         RSS COMMAND                      
             1      630784 init                         
             2           0 kthreadd                     
             3           0 ksoftirqd/0                  
             4           0 events/0                     
             5           0 khelper                      
            50           0 kblockd/0                    
            53           0 kseriod                      
            99           0 pdflush                      
           100           0 pdflush                      
           101           0 kswapd0                      
           102           0 aio/0                        
           103           0 nfsiod                       
           214           0 hid_compat                   
           215           0 rpciod/0                     
           240      614400 udevd                        
           272      577536 udevd                        
           277      565248 udevd                        
           448      524288 sh                           
           449      540672 lina_monitor                 
           450           0 lina_monitor                 
           451    54280192 lina                         
           452           0 lina                         
           453           0 lina                         
           454           0 lina                         
    group "privileged"
      cpu.shares: 16384
      cpuacct.usage: 776952528547140   (100%)
      tasks:
           PID         RSS COMMAND                      
           455           0 lina                         
    group "restricted"
      cpu.shares: 1024
      cpuacct.usage: 1291957168   (0%)
      tasks:
           PID         RSS COMMAND                      
           421     1638400 lwsmd                        
           422           0 lwsmd                        
           423     3686400 lwregd                       
           425           0 lwregd                       
           426           0 lwregd                       
           427           0 lwregd                       
           428           0 lwregd                       
           429           0 lwregd                       
           430           0 lwsmd                        
           431           0 lwsmd                        
           432           0 lwsmd                        
           433           0 lwsmd                        
           434           0 lwsmd                        
    ------------------ show traffic ------------------
    inside:
    received (in 422169.300 secs):
    4183910 packets          523687951 bytes
    9 pkts/sec          1006 bytes/sec
    transmitted (in 422169.300 secs):
    5702974 packets          5851550584 bytes
    3 pkts/sec          13006 bytes/sec
          1 minute input rate 22 pkts/sec,  2839 bytes/sec
          1 minute output rate 30 pkts/sec,  22751 bytes/sec
          1 minute drop rate, 0 pkts/sec
          5 minute input rate 33 pkts/sec,  3746 bytes/sec
          5 minute output rate 46 pkts/sec,  20906 bytes/sec
          5 minute drop rate, 1 pkts/sec
    outside:
    received (in 422169.300 secs):
    10542135 packets          11433861540 bytes
    4 pkts/sec          27002 bytes/sec
    transmitted (in 422169.300 secs):
    3793870 packets          526596330 bytes
    8 pkts/sec          1003 bytes/sec
          1 minute input rate 47 pkts/sec,  41657 bytes/sec
          1 minute output rate 18 pkts/sec,  2802 bytes/sec
          1 minute drop rate, 0 pkts/sec
          5 minute input rate 80 pkts/sec,  38519 bytes/sec
          5 minute output rate 29 pkts/sec,  3749 bytes/sec
          5 minute drop rate, 0 pkts/sec
    _internal_loopback:
    received (in 422168.950 secs):
    0 packets          0 bytes
    0 pkts/sec          0 bytes/sec
    transmitted (in 422168.950 secs):
    0 packets          0 bytes
    0 pkts/sec          0 bytes/sec
          1 minute input rate 0 pkts/sec,  0 bytes/sec
          1 minute output rate 0 pkts/sec,  0 bytes/sec
          1 minute drop rate, 0 pkts/sec
          5 minute input rate 0 pkts/sec,  0 bytes/sec
          5 minute output rate 0 pkts/sec,  0 bytes/sec
          5 minute drop rate, 0 pkts/sec
    Aggregated Traffic on Physical Interface
    Ethernet0/0:
    received (in 776992.730 secs):
    8257731 packets          9051312645 bytes
    5 pkts/sec          11002 bytes/sec
    transmitted (in 776992.730 secs):
    6399342 packets          1011145708 bytes
    2 pkts/sec          1002 bytes/sec
          1 minute input rate 26 pkts/sec,  24481 bytes/sec
          1 minute output rate 20 pkts/sec,  3472 bytes/sec
          1 minute drop rate, 0 pkts/sec
          5 minute input rate 40 pkts/sec,  20147 bytes/sec
          5 minute output rate 29 pkts/sec,  4280 bytes/sec
          5 minute drop rate, 0 pkts/sec
    Ethernet0/1:
    received (in 776992.730 secs):
    1330771 packets          312271947 bytes
    1 pkts/sec          3 bytes/sec
    transmitted (in 776992.730 secs):
    1738316 packets          638003030 bytes
    2 pkts/sec          3 bytes/sec
          1 minute input rate 4 pkts/sec,  405 bytes/sec
          1 minute output rate 11 pkts/sec,  3333 bytes/sec
    <--- More --->
          1 minute drop rate, 0 pkts/sec
          5 minute input rate 7 pkts/sec,  735 bytes/sec
          5 minute output rate 13 pkts/sec,  4410 bytes/sec
          5 minute drop rate, 0 pkts/sec
    Ethernet0/2:
    received (in 776993.220 secs):
    5028958 packets          693527818 bytes
    0 pkts/sec          2 bytes/sec
    transmitted (in 776993.220 secs):
    7782202 packets          8316039741 bytes
    4 pkts/sec          10000 bytes/sec
          1 minute input rate 1 pkts/sec,  153 bytes/sec
          1 minute output rate 2 pkts/sec,  391 bytes/sec
          1 minute drop rate, 0 pkts/sec
          5 minute input rate 1 pkts/sec,  187 bytes/sec
          5 minute output rate 3 pkts/sec,  1011 bytes/sec
          5 minute drop rate, 0 pkts/sec
    Ethernet0/3:
    received (in 776993.220 secs):
    17219822 packets          21609826615 bytes
    0 pkts/sec          27005 bytes/sec
    transmitted (in 776993.220 secs):
    8373382 packets          5142266559 bytes
    5 pkts/sec          6004 bytes/sec
    <--- More --->
          1 minute input rate 8384 pkts/sec,  12695156 bytes/sec
          1 minute output rate 2657 pkts/sec,  203156 bytes/sec
          1 minute drop rate, 0 pkts/sec
          5 minute input rate 8010 pkts/sec,  12112337 bytes/sec
          5 minute output rate 2525 pkts/sec,  188122 bytes/sec
          5 minute drop rate, 0 pkts/sec
    Ethernet0/4:
    received (in 776993.680 secs):
    0 packets          0 bytes
    0 pkts/sec          0 bytes/sec
    transmitted (in 776993.680 secs):
    0 packets          0 bytes
    0 pkts/sec          0 bytes/sec
          1 minute input rate 0 pkts/sec,  0 bytes/sec
          1 minute output rate 0 pkts/sec,  0 bytes/sec
          1 minute drop rate, 0 pkts/sec
          5 minute input rate 0 pkts/sec,  0 bytes/sec
          5 minute output rate 0 pkts/sec,  0 bytes/sec
          5 minute drop rate, 0 pkts/sec
    Ethernet0/5:
    received (in 776993.690 secs):
    0 packets          0 bytes
    0 pkts/sec          0 bytes/sec
    transmitted (in 776993.690 secs):
    <--- More --->
    0 packets          0 bytes
    0 pkts/sec          0 bytes/sec
          1 minute input rate 0 pkts/sec,  0 bytes/sec
          1 minute output rate 0 pkts/sec,  0 bytes/sec
          1 minute drop rate, 0 pkts/sec
          5 minute input rate 0 pkts/sec,  0 bytes/sec
          5 minute output rate 0 pkts/sec,  0 bytes/sec
          5 minute drop rate, 0 pkts/sec
    Ethernet0/6:
    received (in 776994.140 secs):
    0 packets          0 bytes
    0 pkts/sec          0 bytes/sec
    transmitted (in 776994.140 secs):
    0 packets          0 bytes
    0 pkts/sec          0 bytes/sec
          1 minute input rate 0 pkts/sec,  0 bytes/sec
          1 minute output rate 0 pkts/sec,  0 bytes/sec
          1 minute drop rate, 0 pkts/sec
          5 minute input rate 0 pkts/sec,  0 bytes/sec
          5 minute output rate 0 pkts/sec,  0 bytes/sec
          5 minute drop rate, 0 pkts/sec
    Ethernet0/7:
    received (in 776994.140 secs):
    7328915 packets          4524298170 bytes
    <--- More --->
    3 pkts/sec          5004 bytes/sec
    transmitted (in 776994.140 secs):
    16345245 packets          21405489647 bytes
    4 pkts/sec          27001 bytes/sec
          1 minute input rate 2330 pkts/sec,  158045 bytes/sec
          1 minute output rate 7422 pkts/sec,  11264540 bytes/sec
          1 minute drop rate, 0 pkts/sec
          5 minute input rate 2481 pkts/sec,  168427 bytes/sec
          5 minute output rate 7977 pkts/sec,  12105867 bytes/sec
          5 minute drop rate, 0 pkts/sec
    Internal-Data0/0:
    received (in 776994.640 secs):
    15222548 packets          10134365294 bytes
    3 pkts/sec          13004 bytes/sec
    transmitted (in 776994.640 secs):
    15128813 packets          10256961010 bytes
    2 pkts/sec          13001 bytes/sec
          1 minute input rate 45 pkts/sec,  24860 bytes/sec
          1 minute output rate 49 pkts/sec,  26647 bytes/sec
          1 minute drop rate, 0 pkts/sec
          5 minute input rate 73 pkts/sec,  24918 bytes/sec
          5 minute output rate 75 pkts/sec,  26334 bytes/sec
          5 minute drop rate, 0 pkts/sec
    Internal-Data0/1:
    <--- More --->
    received (in 776994.640 secs):
    15128721 packets          10256943282 bytes
    2 pkts/sec          13001 bytes/sec
    transmitted (in 776994.640 secs):
    15222455 packets          10134357062 bytes
    3 pkts/sec          13004 bytes/sec
          1 minute input rate 48 pkts/sec,  26530 bytes/sec
          1 minute output rate 45 pkts/sec,  24826 bytes/sec
          1 minute drop rate, 0 pkts/sec
          5 minute input rate 75 pkts/sec,  26323 bytes/sec
          5 minute output rate 73 pkts/sec,  24908 bytes/sec
          5 minute drop rate, 0 pkts/sec
    ------------------ show perfmon ------------------
    PERFMON STATS:                     Current      Average
    Xlates                                0/s          0/s
    Connections                           0/s          0/s
    TCP Conns                             0/s          0/s
    UDP Conns                             0/s          0/s
    URL Access                            0/s          0/s
    URL Server Req       

Maybe you are looking for

  • Do i have to send a confirmation that i a m a student every semester or is my option safe for a year?

    hi, i have a CC student subscription in germany for 19,99€ a month. my time as a student either ends at the end of march 2015 or - if i must prolong it for several reasons  - will end in october 2015. my contract with adobe ends in june 2015. so what

  • Can't Start WL 6.1sp1 on Win2000sp1

    I can't start WL 6.1sp1 on my win2000 laptop, but WL6.0 works. for some reason it can't find weblogic.server. I tried several re-installs no luck. Class path look good my classpath is weblogic\bea\wlserver6.1>set CLASSPATH=.;.c:\weblogic\bea\jdk131\l

  • How to clear apps from notification center?

    Is there a way to reset/clear apps from notification center? My facebook notification ain't working and i want to reset notification center.

  • Indexes created but shows as missing

    Hi Gurus, I have an issue ,where the system log shows that indexes are successfully created from drop indexes after successful data load. but there are missing secondary indexes existing for the same cube in DB02. There are two process chains based o

  • Reversal on Import Duty Provision posted thru MIGO

    Hi All, I have a issue on reversal of Import Duty provision..when GR is done for Imports the Import Duty Provision is posted to GL..there are some old provisions in this GL which needs to be reversed because there is no requirement of these provision